Skip to main content
Log in

Global Hilbert Expansion for the Relativistic Vlasov–Maxwell–Boltzmann System

  • Published:
Communications in Mathematical Physics Aims and scope Submit manuscript

Abstract

Consider the relativistic Vlasov–Maxwell–Boltzmann system describing the dynamics of an electron gas in the presence of a fixed ion background. Thanks to recent works Germain and Masmoudi (Ann Sci Éc Norm Supér 47(3):469–503, 2014), Guo et al. (J Math Phys 55(12):123102, 2014) and Deng et al. (Arch Ration Mech Anal 225(2):771–871, 2017), we establish the global-in-time validity of its Hilbert expansion and derive the limiting relativistic Euler–Maxwell system as the mean free path goes to zero. Our method is based on the \(L^2-L^{\infty }\) framework and the Glassey–Strauss Representation of the electromagnetic field, with auxiliary \(H^1\) estimates and \(W^{1,\infty }\) estimates to control the characteristic curves and corresponding \(L^{\infty }\) norm.

This is a preview of subscription content, log in via an institution to check access.

Access this article

Price excludes VAT (USA)
Tax calculation will be finalised during checkout.

Instant access to the full article PDF.

Similar content being viewed by others

References

  1. Arsénio, D., Saint-Raymond, L.: Solutions of the Vlasov–Maxwell–Boltzmann system with long-range interactions. C. R. Math. Acad. Sci. Paris 351(9–10), 357–360 (2013)

    Article  MathSciNet  Google Scholar 

  2. Arsénio, D., Saint-Raymond, L.: From the Vlasov–Maxwell–Boltzmann system to incompressible viscous electro-magneto-hydrodynamics. Vol. 1. EMS Monographs in Mathematics. European Mathematical Society (EMS), Zürich, 2019. xii+406 pp

  3. Bahouri, H., Chemin, J. Y., Danchin, R.: Fourier Analysis and Nonlinear Partial Differential Equations. Grundlehren der mathematischen Wissenschaften, vol. 343. Springer, Berlin (2011)

  4. Caflisch, R.E.: The fluid dynamic limit of the nonlinear Boltzmann equation. Commun. Pure Appl. Math. 33(5), 651–666 (1980)

    Article  ADS  MathSciNet  Google Scholar 

  5. Christodoulou, D.: The Formation of Shocks in 3-Dimensional Fluids. European Mathematical Society, Zürich (2007)

    Book  Google Scholar 

  6. Deng, Y., Ionescu, A.D., Pausader, B.: The Euler–Maxwell system for electrons: global solutions in 2D. Arch. Ration. Mech. Anal. 225(2), 771–871 (2017)

    Article  MathSciNet  Google Scholar 

  7. Duan, R.J., Lei, Y.J., Yang, T., Zhao, H.J.: The Vlasov–Maxwell–Boltzmann system near Maxwellians in the whole space with very soft potentials. Commun. Math. Phys. 351(1), 95–153 (2017)

    Article  ADS  MathSciNet  Google Scholar 

  8. Duan, R.J., Liu, S.Q., Yang, T., Zhao, H.J.: Stability of the nonrelativistic Vlasov–Maxwell–Boltzmann system for angular non-cutoff potentials. Kinetic Related Models 6(1), 159–204 (2013)

    Article  MathSciNet  Google Scholar 

  9. Duan, R.J., Strain, R.M.: Optimal large-time behavior of the Vlasov–Maxwell–Boltzmann system in the whole space. Commun. Pure Appl. Math. 64(11), 1497–1546 (2011)

    MathSciNet  MATH  Google Scholar 

  10. Dudyński, Marek, Ekiel-Jeżewska, M.L.: On the linearized relativistic Boltzmann equation. I. Existence of solutions. Commun. Math. Phys. 115(4), 607–629 (1988)

    Article  ADS  MathSciNet  Google Scholar 

  11. Dudyńnski, M., Ekiel-Jeżewska, M.L.: The relativistic Boltzmann equation-mathematical and physical aspects. J. Tech. Phys. 48(1), 39–47 (2007)

  12. Germain, P., Masmoudi, N.: Global existence for the Euler–Maxwell system. Ann. Sci. Éc. Norm. Supér. 47(3), 469–503 (2014)

    Article  MathSciNet  Google Scholar 

  13. Glassey, R.T., Schaeffer, J.: The two and one-half-dimensional” relativistic Vlasov Maxwell system. Commun. Math. Phys. 185(2), 257–284 (1997)

    Article  ADS  MathSciNet  Google Scholar 

  14. Glassey, R.T., Strauss, W.A.: Singularity formation in a collisionless plasma could occur only at high velocities. Arch. Rational Mech. Anal. 92(1), 59–90 (1986)

    Article  ADS  MathSciNet  Google Scholar 

  15. Glassey, R.T., Strauss, W.A.: On the derivatives of the collision map of relativistic particles. Transp. Theory Stat. Phys. 20(1), 55–68 (1991)

    Article  ADS  MathSciNet  Google Scholar 

  16. Glassey, R.T., Strauss, W.A.: Asymptotic stability of the relativistic Maxwellian. Publ. Res. Inst. Math. Sci. 29(2), 301–347 (1993)

    Article  MathSciNet  Google Scholar 

  17. Guo, Y.: Smooth irrotational flows in the large to the Euler–Poisson system in \({\mathbb{R}}^{3+1}\). Commun. Math. Phys. 195(2), 249–265 (1998)

    Article  ADS  Google Scholar 

  18. Guo, Y.: The Vlasov–Maxwell–Boltzmann system near Maxwellians. Invent. Math. 153(3), 593–630 (2003)

    Article  ADS  MathSciNet  Google Scholar 

  19. Guo, Y.: Decay and continuity of the Boltzmann equation in bounded domains. Arch. Ration. Mech. Anal. 197(3), 713–809 (2010)

    Article  MathSciNet  Google Scholar 

  20. Guo, Y., Ionescu, A.D., Pausader, B.: Global solutions of certain plasma fluid models in three-dimension. J. Math. Phys. 55(12), 123102,26 (2014)

    Article  MathSciNet  Google Scholar 

  21. Guo, Y., Ionescu, Alexandru D.: Pausader, Benoit Global solutions of the Euler–Maxwell two-fluid system in \(3D\). Ann. Math. 183(2), 377–498 (2016)

    Article  MathSciNet  Google Scholar 

  22. Guo, Y., Jang, J.: Global Hilbert expansion for the Vlasov–Poisson–Boltzmann system. Commun. Math. Phys. 299(2), 469–501 (2010)

    Article  ADS  MathSciNet  Google Scholar 

  23. Guo, Y., Jang, J., Jiang, N.: Local Hilbert expansion for the Boltzmann equation. Kinet. Relat. Models 2(1), 205–214 (2009)

    Article  MathSciNet  Google Scholar 

  24. Guo, Y., Jang, J., Jiang, N.: Acoustic limit for the Boltzmann equation in optimal scaling. Commun. Pure Appl. Math. 63(3), 337–361 (2010)

    MathSciNet  MATH  Google Scholar 

  25. Guo, Y., Strain, R.M.: Momentum regularity and stability of the relativistic Vlasov–Maxwell–Boltzmann system. Commun. Math. Phys. 310(3), 649–673 (2012)

    Article  ADS  MathSciNet  Google Scholar 

  26. Ha, S.-Y., Xiao, Q.H., Xiong, L.J., Zhao, H.J.: \(L^2\)-stability of the Vlasov–Maxwell–Boltzmann system near global Maxwellians. J. Math. Phys. 54(12), 121509, 17 (2013)

    Article  Google Scholar 

  27. Jang, J.: Vlasov–Maxwell–Boltzmann diffusive limit. Arch. Ration. Mech. Anal. 194(2), 531–584 (2009)

    Article  MathSciNet  Google Scholar 

  28. Lei, Y.J., Zhao, H.J.: The Vlasov–Maxwell–Boltzmann system with a uniform ionic background near Maxwellians. J. Differ. Equ. 260(3), 2830–2897 (2016)

    Article  ADS  MathSciNet  Google Scholar 

  29. Liu, S.Q., Xiao, Q.H.: The relativistic Vlasov–Maxwell–Boltzmann system for short range interaction. Kinet. Relat. Models 9(3), 515–550 (2016)

    Article  MathSciNet  Google Scholar 

  30. Ma, F.H., Ma, X.: Stability of the relativistic Vlasov–Maxwell–Boltzmann system for short range interaction. Appl. Math. Comput. 265, 854–882 (2015)

    MathSciNet  MATH  Google Scholar 

  31. Ruggeri, T., Xiao, Q.H., Zhao, H.J.: The Riemann problem of relativistic Euler system with Synge energy. Arch. Ration. Mech. Anal. 239(2), 1061–1109 (2021). arXiv:abs/2001.04128v1 [math-ph] 2020

  32. Sideris, T.C.: Formation of singularities in three-dimensional compressible fluids. Commun. Math. Phys. 101(4), 475–485 (1985)

    Article  ADS  MathSciNet  Google Scholar 

  33. Speck, J., Strain, R.M.: Hilbert expansion from the Boltzmann equation to relativistic fluids. Commun. Math. Phys. 304(1), 229–280 (2011)

    Article  ADS  MathSciNet  Google Scholar 

  34. Strain, R.M.: Some Applications of an Energy Method in Collisional Kinetic Theory. Brown University, Providence RI (2005). Phd Thesis

  35. Strain, R.M.: The Vlasov–Maxwell–Boltzmann system in the whole space. Commun. Math. Phys. 268(2), 543–567 (2006)

    Article  ADS  MathSciNet  Google Scholar 

  36. Strain, R.M.: Global Newtonian limit for the relativistic Boltzmann equation near vacuum. SIAM J. Math. Anal. 42(4), 1568–1601 (2010). well-posedness of characteristic symmetric hyperbolic systems

  37. Strain, R.M.: Asymptotic stability of the relativistic Boltzmann equation for the soft-potentials. Commun. Math. Phys. 300(2), 529–597 (2010)

    Article  ADS  MathSciNet  Google Scholar 

Download references

Acknowledgements

The work of Yan Guo is supported by NSF Grant DMS-1810868, the work of Qinghua Xiao is supported by grants from Youth Innovation Promotion Association, CAS (No. 2017379), the National Natural Science Foundation of China under contract 11871469, the State Scholarship Fund of China and Hubei Chenguang Talented Youth Development Foundation.

Author information

Authors and Affiliations

Authors

Corresponding author

Correspondence to Yan Guo.

Additional information

Communicated by A. Ionescu.

Publisher's Note

Springer Nature remains neutral with regard to jurisdictional claims in published maps and institutional affiliations.

Appendix

Appendix

Our Appendix is about three problems: the derivation of the kernel of operator \(K_{{\mathbf {M}}}\) in (1.21), estimates of the kernels related to \(K_1\) and \(K_2\), and the construction and regularity estimates of the coefficients in the Hilbert expansion (1.8).

1.1 Appendix 1: Derivation of the kernel \(k_2\)

In this part, we derive the kernel \(k_2\) of the operator \(K_2\). For \(K_i, (i=1, 2)\) given in (1.21), their kernels are defined as

$$\begin{aligned} K_i(f)=\int _{{\mathbb {R}}^3}dq k_i(p,q)f(q),\qquad i=1,2. \end{aligned}$$

Following the derivation of \(k_2(p,q)\) for a global Maxwellian case in the Appendix of [37], we can obtain the following expression of \(k_2(p,q)\) for our case:

$$\begin{aligned} k_2(p,q)=\frac{C_1s^{\frac{3}{2}}}{gp^0q^0}U_1(p,q)\exp \{-U_2(p,q)\}, \end{aligned}$$
(8.1)

where \(C_1\) is some positive constant, \(U_2(p,q)\) and \(U_1(p,q)\) are smooth functions satisfying

$$\begin{aligned} U_2(p,q)&=[1+O(1)|u|]\frac{\sqrt{s}|p-q|}{2T_0g},\\ U_1(p,q)&=\frac{1}{U_2(p,q)}+\frac{(p^0+q^0)[1+O(1)|u|]}{2[U_2(p,q)]^2}+\frac{(p^0+q^0)[1+O(1)|u|]}{2[U_2(p,q)]^3}. \end{aligned}$$

The proof of (8.1) can be proceeded similarly as in the Appendix [37] except for the estimation of additional terms w.r.t. the velocity u, which arises due to the local Maxwellian \(\mathbf {M}\).

As in [37], we first write the operator \(K_2\) as

$$\begin{aligned} K_2(f)=\frac{1}{p^0}\int _{{\mathbb {R}}^3}\frac{dq}{q^0}\int _{{\mathbb {R}}^3}\frac{dq'}{q'^{0}}\int _{{\mathbb {R}}^3}\frac{dp'}{p'^{0}}W\sqrt{{\mathbf {M}}}(q)[\sqrt{{\mathbf {M}}}(q')f(p')+\sqrt{{\mathbf {M}}}(p')f(q')]. \end{aligned}$$

After the same exchanges of variables \(q, p', q'\) and changes of integration variables as in [37], we obtain the following form of kernel \(k_2\)

$$\begin{aligned} k_2(p,q)=\frac{C}{p^0q^{0}}\int _{{\mathbb {R}}^3}\frac{d{\bar{p}}}{{\bar{p}}^0}\delta ((q^{\mu }-p^{\mu }){\bar{p}}_{\mu }){\bar{s}} \exp \Big \{\frac{u^{\mu }{\bar{p}}_{\mu } }{2T_0}\Big \}, \end{aligned}$$

where the integration variable \({\bar{p}}_{\mu }=p'_{\mu }+q'_{\mu }\) and \({\bar{s}}={\bar{g}}^2+4\) with

$$\begin{aligned} {\bar{g}}^2=g^2+\frac{1}{2}(p^{\mu }+q^{\mu })(p_{\mu }+q_{\mu }-{\bar{p}}_{\mu }). \end{aligned}$$

Introduce a Lorentz transformation \(\Lambda \) which maps into the center-of-momentum system:

$$\begin{aligned} A^{\mu }=\Lambda ^{\mu }_{\nu }(p^{\nu }+q^{\nu })=(\sqrt{s},0,0,0), \quad B^{\mu }=-\Lambda ^{\mu }_{\nu }(p^{\nu }-q^{\nu })=(0,0,0,g). \end{aligned}$$

Here \(\Lambda \) is the following Lorentz transform derived in [34]:

$$\begin{aligned} \Lambda =(\Lambda ^{\mu }_{\nu })=\left( \begin{array}{cccc} \frac{p^0+q^0}{\sqrt{s}} &{} -\frac{p_1+q_1}{\sqrt{s}} &{} -\frac{p_2+q_2}{\sqrt{s}} &{}-\frac{p_3+q_3}{\sqrt{s}}\\ \Lambda ^{1}_0 &{} \Lambda ^{1}_{1} &{} \Lambda ^{1}_{2} &{} \Lambda ^{1}_3 \\ 0 &{} \frac{(p\times q)_1}{|p\times q|} &{} \frac{(p\times q)_2}{|p\times q|} &{} \frac{(p\times q)_3}{|p\times q|}\\ \frac{p^0-q^0}{g} &{} -\frac{p_1-q_1}{g} &{} -\frac{p_2-q_2}{g} &{}-\frac{p_3-q_3}{g} \end{array} \right) , \end{aligned}$$

where \(\Lambda ^{1}_0=\frac{2|p\times q|}{g\sqrt{s}}\) and

$$\begin{aligned} \Lambda ^{1}_i=\frac{2[p_i(p^0+q^0p^{\mu }q_{\mu })+q_i(q^0+p^0p^{\mu }q_{\mu })]}{g\sqrt{s}|p\times q|}, \quad i=1,2,3. \end{aligned}$$

Define \({\bar{u}}^\mu =\Lambda ^{\mu }_{\nu }u^{\nu }\):

$$\begin{aligned} {\bar{u}}^0&=\Lambda ^{0}_{\nu }u^{\nu }=\frac{(p^0+q^0)u^0}{\sqrt{s}} -\frac{(p+q)\cdot u}{\sqrt{s}},\\ {\bar{u}}^1&=\Lambda ^{1}_{\nu }u^{\nu }=\frac{2|p\times q|u^0}{g\sqrt{s}} +\frac{2[p(p^0+q^0p^{\mu }q_{\mu })+q(q^0+p^0p^{\mu }q_{\mu })]\cdot u}{g\sqrt{s}|p\times q|},\\ {\bar{u}}^2&=\Lambda ^{2}_{\nu }u^{\nu }=\frac{(p\times q)\cdot u}{|p\times q|},\\ {\bar{u}}^3&=\Lambda ^{3}_{\nu }u^{\nu }=\frac{(p^0-q^0)u^0}{g} -\frac{(p-q)\cdot u}{g}. \end{aligned}$$

Then we have

$$\begin{aligned} \int _{{\mathbb {R}}^3}\frac{d{\bar{p}}}{{\bar{p}}^0}\delta ((q^{\mu }-p^{\mu }){\bar{p}}_{\mu }){\bar{s}} \exp \Big \{\frac{u^{\mu }{\bar{p}}_{\mu } }{2T_0}\Big \}=\int _{{\mathbb {R}}^3}\frac{d{\bar{p}}}{{\bar{p}}^0}\delta (B^{\mu }{\bar{p}}_{\mu }){\bar{s}}_{\Lambda } \exp \Big \{\frac{u^{\mu }{\bar{p}}_{\mu } }{2T_0}\Big \}, \end{aligned}$$

where \(B^{\mu }{\bar{p}}_{\mu }={\bar{p}}_3g,\)

$$\begin{aligned} {\bar{s}}_{\Lambda }&={\bar{g}}^2+4=g^2+\frac{1}{2}A^{\mu }(A_{\mu }-{\bar{p}}_{\mu })=g^2+\frac{1}{2}\sqrt{s}({\bar{p}}^0-\sqrt{s}),\\ {\bar{u}}^{\mu }{\bar{p}}_{\mu }=&{\bar{p}}^0\Big (-\frac{(p^0+q^0)u^0}{\sqrt{s}} +\frac{(p+q)\cdot u}{\sqrt{s}}\Big )\\&\quad +{\bar{p}}_1\Big (\frac{2|p\times q|u^0}{g\sqrt{s}} +\frac{2[p(p^0+q^0p^{\mu }q_{\mu })+q(q^0+p^0p^{\mu }q_{\mu })]\cdot u}{g\sqrt{s}|p\times q|}\Big )\\&\quad +{\bar{p}}_2\frac{(p\times q)\cdot u}{|p\times q|}+{\bar{p}}_3\Big (\frac{(p^0-q^0)u^0}{g} -\frac{(p-q)\cdot u}{g}\Big ). \end{aligned}$$

Switch to polar coordinates

$$\begin{aligned} d{\bar{p}}=|{\bar{p}}|^2d|{\bar{p}}|\sin \phi d\phi d\theta ,\quad {\bar{p}}=|{\bar{p}}|(\sin \phi \cos \theta , \sin \phi \sin \theta , \cos \phi ), \end{aligned}$$

and use \(\cos \phi =0\) for \(\phi =\frac{\pi }{2}\) to rewrite \( k_2(p,q)\) as

$$\begin{aligned}&\frac{C}{gp^0q^0}\int _{0}^{2\pi }d\phi \int _0^{\infty }\frac{|{\bar{p}}|d{\bar{p}}}{{\bar{p}}^0}{\bar{s}}_{\Lambda } \exp \Big \{\frac{1}{2T_0}\Big [{\bar{p}}^0\Big (-\frac{(p^0+q^0)u^0}{\sqrt{s}} +\frac{(p+q)\cdot u}{\sqrt{s}}\Big )\\&\qquad +\Big (\frac{2|p\times q|u^0}{g\sqrt{s}} +\frac{2[p(p^0+q^0p^{\mu }q_{\mu })+q(q^0+p^0p^{\mu }q_{\mu })]\cdot u}{g\sqrt{s}|p\times q|}\Big )|{\bar{p}}|\cos \phi \\&\qquad +\frac{(p\times q)\cdot u}{|p\times q|}|{\bar{p}}|\sin \phi \Big ] \Big \}\\&\quad =\frac{C}{gp^0q^0}\int _{0}^{\infty }\frac{|{\bar{p}}|d{\bar{p}}}{{\bar{p}}^0}{\bar{s}}_{\Lambda } \exp \Big \{\frac{{\bar{p}}^0}{2T_0}\Big (-\frac{(p^0+q^0)u^0}{\sqrt{s}} +\frac{(p+q)\cdot u}{\sqrt{s}}\Big )\Big \}\\&\qquad \times I_0\Big (\frac{\sqrt{{g^2s|(p\times q)\cdot u|^2+4[|p\times q|^2u^0+(p(p^0+q^0p^{\mu }q_{\mu })+q(q^0+p^0p^{\mu }q_{\mu }))\cdot u]^2}}}{2T_0g\sqrt{s}|p\times q|}|{\bar{p}}|\Big ), \end{aligned}$$

where \(I_0\) is the first kind modified Bessel function of index zero:

$$\begin{aligned} I_0(y)=\frac{1}{2\pi }\int _0^{2\pi }e^{y\cos \phi }d\phi . \end{aligned}$$

By further changing variables of integration and applying some known integrals as in the Appendix [37], we obtain (8.1) with the following exact form of \(U_2(p,q)\):

$$\begin{aligned} 4T_0^2sU_2^2&=|(p^0+q^0)u^0-(p+q)\cdot u|^2-\frac{s|(p\times q)\cdot u|^2}{|p\times q|^2}\nonumber \\&\quad -\frac{4[|p\times q|^2u^0+(p(p^0+q^0p^{\mu }q_{\mu })+q(q^0+p^0p^{\mu }q_{\mu }))\cdot u]^2}{g^2|p\times q|^2}\nonumber \\&=\frac{g^2(p^0+q^0)^2-4|p\times q|^2}{g^2}(u^0)^2+|(p+q)\cdot u|^2-\frac{s|(p\times q)\cdot u|^2}{|p\times q|^2}\nonumber \\&\quad -\frac{2u^0[g^2(p^0+q^0)(p+q)+4(p(p^0+q^0p^{\mu }q_{\mu })+q(q^0+p^0p^{\mu }q_{\mu }))]\cdot u}{g^2}\nonumber \\&\quad -\frac{|(p(p^0+q^0p^{\mu }q_{\mu })+q(q^0+p^0p^{\mu }q_{\mu }))\cdot u|^2}{g^2|p\times q|^2}. \end{aligned}$$
(8.2)

Now we estimate the terms in the right hand side of the second equality in (8.2). For the first term, we have

$$\begin{aligned} \begin{aligned}&g^2(p^0+q^0)^2-4|p\times q|^2\\&\quad =2[(2p^0q^0+2+|p|^2+|q|^2)(p^0q^0-p\cdot q-1)-4|p|^2|q|^2+4(p\cdot q)^2\\&\quad =4[(|p|^2+|q|^2)-(p^0q^0+1)p\cdot q]\\&\qquad +2(|p|^2+|q|^2)(p^0q^0-p\cdot q-1)+4(p\cdot q)^2\\&\quad =2(|p|^2+|q|^2)(p^0q^0-p\cdot q+1)-4p\cdot q(p^0q^0-p\cdot q+1)\\&\quad =s|p-q|^2. \end{aligned} \end{aligned}$$
(8.3)

It is straightforward to see that the upper bound of the second term is \(s|u|^2\). Noting

$$\begin{aligned}&g^2(p^0+q^0)(p+q)+4(p(p^0+q^0p^{\mu }q_{\mu })+q(q^0+p^0p^{\mu }q_{\mu }))\\&\quad =2p[(p^0+q^0)(p^0q^0-p\cdot q-1)-2p^0|q|^2+2q^0p\cdot q)]\\&\qquad +2q[(p^0+q^0)(p^0q^0-p\cdot q-1)-2q^0|p|^2+2p^0p\cdot q)]\\&\quad =2p(p^0-q^0)(p^0q^0-p\cdot q+1)-2q(p^0-q^0)(p^0q^0-p\cdot q+1)\\&\quad =s(p^0-q^0)(p-q), \end{aligned}$$

the fourth term can be bounded by

$$\begin{aligned} \frac{2u^0|u|s|p-q|^2}{g^2}. \end{aligned}$$

For the corresponding fifth term, we rewrite its numerator as

$$\begin{aligned}&|p(p^0+q^0p^{\mu }q_{\mu })+q(q^0+p^0p^{\mu }q_{\mu })|^2\\&\quad =|p|^2[(p^0)^2|q|^4-2p^0q^0|q|^2p\cdot q+(q^0)^2(p\cdot q)^2]\\&\qquad +|q|^2[(q^0)^2|p|^4-2p^0q^0|p|^2p\cdot q+(p^0)^2(p\cdot q)^2]\\&\qquad +2p\cdot q[p^0q^0|p|^2|q|^2-((p^0)^2|q|^2+(q^0)^2|p|^2)p\cdot q+p^0q^0(p\cdot q)^2]\\&\quad =|p|^2|q|^2\left\{ [(p^0)^2|q|^2+(q^0)^2|p|^2+2p^0q^0p\cdot q]-4p^0q^0p\cdot q\right\} \\&\qquad +[|p|^2(q^0)^2+|q|^2(p^0)^2-2(|q|^2(p^0)^2+|p|^2(q^0)^2)](p\cdot q)^2+2p^0q^0(p\cdot q)^3\\&\quad =|p|^2|q|^2|p^0q-q^0p|^2-(p\cdot q)^2[(p^0)^2|q|^2+(q^0)^2|p|^2-2p^0q^0(p\cdot q)]\\&\quad =|p\times q|^2|p^0q-q^0p|^2. \end{aligned}$$

Then it can be bounded by

$$\begin{aligned} \frac{|u|^2|p\times q|^2|p^0q-q^0p|^2}{g^2|p\times q|^2}=\frac{|u|^2|p^0q-q^0p|^2}{g^2}. \end{aligned}$$

On the other hand, we have

$$\begin{aligned}&s|p- q|^2-|p^0q-q^0p|^2\\&\quad =2(p^0q^0-p\cdot q+1)(|p|^2+|q|^2-2p\cdot q)-|q|^2(p^0)^2-|p|^2(q^0)^2+2p^0q^0p\cdot q\\&\quad =4(p\cdot q)^2-2[(p^0)^2+(q^0)^2+p^0q^0]p\cdot q\\&\qquad +2(|p|^2+|q|^2)(p^0q^0+1)-[(p^0)^2|q|^2+(q^0)^2|p|^2]\\&\qquad \ge 4|p|^2|q|^2-(2|p|^2+2|q|^2+2p^0q^0+4)|p||q|\\&\qquad +2(|p|^2+|q|^2)p^0q^0+|p|^2+|q|^2-2|p|^2|q|^2\\&\quad =2(|p|^2+|q|^2-|p||q|)(p^0q^0-|p||q|)+(|p|^2+|q|^2-4|p||q|)\\&\quad \ge 3(|p|-|q|)^2 \end{aligned}$$

since \(p^0q^0-|p||q|\ge 1.\) We combine the above estimates in (8.2) to get

$$\begin{aligned} 4T_0^2U_2^2(p,q)=[1+O(1)|u|]^2\frac{s|p-q|^2}{g^2}. \end{aligned}$$

1.2 Appendix 2: Estimates of kernels

We first prove some important estimates for kernels \(k_1\) and \(k_2\). As a corollary, estimates for corresponding kernels of \({\bar{K}}(h^{\varepsilon })=\frac{w(|p|)\sqrt{\mathbf{M}}}{\sqrt{J_{M}}}K(\frac{\sqrt{J_{M}}}{w\sqrt{\mathbf{M}}}h^{\varepsilon })\) will also be established.

Lemma 8.1

For \(k_2\) given in (8.1) and \(i=1,2,3\), it holds that

$$\begin{aligned} k_2(p,q)\lesssim \frac{1}{p^0|p-q|}\exp \Big \{-\frac{\sqrt{s}|p-q|}{8T_0g}\Big \}, \end{aligned}$$
(8.4)

and

$$\begin{aligned}&\int _{{\mathbb {R}}^3}dq k_2(p,q)\lesssim \frac{1}{p^0},\nonumber \\&\quad \int _{{\mathbb {R}}^3}dq \partial _{x_i} k_2(p,q)\lesssim \frac{1}{p^0},\nonumber \\&\quad \int _{{\mathbb {R}}^3}dq \partial _{p_i} k_2(p,q)\lesssim \frac{1}{p^0}. \end{aligned}$$
(8.5)

It is straightforward to verify that \(k_1(p,q)\) also satisfies the same estimates.

Proof

We first prove (8.4). Note that, by Lemma 3.1 in [16], \(s=g^2+4\le 4p^0q^0+4\) and

$$\begin{aligned} \frac{|p-q|}{\sqrt{p^0q^0}}\le g\le |p-q|. \end{aligned}$$

We use the smallness assumption of |u| in (1.20) to have

$$\begin{aligned} U_2(p,q)\ge&\frac{\sqrt{s}|p-q|}{4T_0g},\qquad U_1(p,q)\lesssim \frac{p^0+q^0}{2[U_2(p,q)]^2}. \end{aligned}$$

Then we can further estimate the kernel \(k_2\) as

$$\begin{aligned} k_2(p,q)&\lesssim \frac{s^{\frac{3}{2}}}{gp^0q^0}(p^0+q^0)\times \frac{g^2}{s|p-q|^2}\exp \{-\frac{\sqrt{s}|p-q|}{4T_0g}\}\\&= \frac{(p^0+q^0)g^2}{p^0q^0|p-q|^3}\frac{\sqrt{s}|p-q|}{g}\exp \{-\frac{\sqrt{s}|p-q|}{4T_0g}\}\\&\lesssim \frac{1+|p-q|+q^0}{p^0q^0|p-q|}\exp \{-\frac{|p-q|}{6T_0}\}\\&\lesssim \frac{1}{p^0|p-q|}\exp \{-\frac{|p-q|}{8T_0}\}. \end{aligned}$$

Now we prove (8.5). Noting that

$$\begin{aligned} |p|^2\le 2|p-q|^2+2|q|^2,\qquad |q|^2\le 2|p-q|^2+2|p|^2, \end{aligned}$$

we get from (8.4) that

$$\begin{aligned} \int _{{\mathbb {R}}^3}dq k_2(p,q)\lesssim&\int _{{\mathbb {R}}^3}dq\frac{1}{p^0|p-q|}\exp \Big \{-\frac{|p-q|}{8T_0}\Big \}\lesssim \frac{1}{p^0}. \end{aligned}$$

For the second inequality in (8.5), again from (8.4), we obtain

$$\begin{aligned}&\int _{{\mathbb {R}}^3}dq \partial _{x_i} k_2(p,q)\\&\quad \lesssim \int _{{\mathbb {R}}^3}dq k_2(p,q) \frac{\sqrt{s}|p-q|}{g} \exp \{-\frac{\sqrt{s}|p-q|}{4T_0g}\}\\&\quad \lesssim \int _{{\mathbb {R}}^3}dq k_2(p,q) \exp \{-\frac{\sqrt{s}|p-q|}{8T_0g}\}\\&\quad \lesssim \frac{1}{p^0}. \end{aligned}$$

Now we prove the third inequality in (8.5). Note that

$$\begin{aligned} \partial _{p_i}U_2(p,q)&=[1+O(1)|u|]\partial _{p_i}\Big (\frac{\sqrt{s}|p-q|}{2T_0g}\Big )\\&=[1+O(1)|u|]\Big (\frac{|p-q|\partial _{p_i}s}{4T_0g\sqrt{s}}+\frac{\sqrt{s}(p_i-q_i)}{2T_0g|p-q|} -\frac{\sqrt{s}|p-q|\partial _{p_i}g}{2T_0g^2}\Big )\\&=[1+O(1)|u|]\Big [\frac{-4|p-q|}{2T_0g^2\sqrt{s}}\frac{q^0}{g}\Big (\frac{p_i}{p^0}-\frac{q_i}{q^0}\Big ) +\frac{\sqrt{s}(p_i-q_i)}{2T_0g|p-q|}\Big ]\\&\lesssim \frac{s^{\frac{3}{2}}|p-q|^3}{g^3} \frac{q^0}{s^2|p-q|\min \{p_0,q_0\}}+\frac{\sqrt{s}|p-q|}{2T_0g}\frac{1}{|p-q|}\\&\lesssim \frac{s^{\frac{3}{2}}|p-q|^3}{g^3} \frac{\min \{p_0,q_0\}+|p-q|}{|p-q|\min \{p_0,q_0\}}+\frac{\sqrt{s}|p-q|}{2T_0g}\frac{1}{|p-q|}, \end{aligned}$$

where we have used the following inequality:

$$\begin{aligned} \frac{p_i}{p^0}-\frac{q_i}{q^0}\le \frac{|p-q|}{\min \{p_0,q_0\}}. \end{aligned}$$

Then we use (8.4) to obtain

$$\begin{aligned}&\int _{{\mathbb {R}}^3}dq \partial _{p_i} k_2(p,q)\\&\quad \lesssim \int _{{\mathbb {R}}^3}dq \Big (\frac{s^{\frac{3}{2}}|p-q|^3}{g^3} \frac{\min \{p_0,q_0\}+|p-q|}{|p-q|\min \{p_0,q_0\}}+\frac{\sqrt{s}|p-q|}{2T_0g}\frac{1}{|p-q|}\Big )k_2(p,q)\\&\quad \lesssim \int _{{\mathbb {R}}^3}dq \frac{1}{p^0}\Big (\frac{s^{\frac{3}{2}}|p-q|^3}{g^3}+1\Big ) \Big (1+\frac{1}{|p-q|^2}\Big )\exp \Big \{-\frac{\sqrt{s}|p-q|}{8T_0g}\Big \}\\&\quad \lesssim \int _{{\mathbb {R}}^3}dq \frac{1}{p^0}\Big (1+\frac{1}{|p-q|^2}\Big )\exp \Big \{-\frac{\sqrt{s}|p-q|}{16T_0g}\Big \}\\&\quad \lesssim \frac{1}{p^0}. \end{aligned}$$

\(\square \)

According to the definition of \(h^{\varepsilon }\) in (1.23), the operator K corresponding to the equation satisfied by \(h^{\varepsilon }\) is \({\bar{K}}(h^{\varepsilon })=\frac{w(|p|)\sqrt{\mathbf{M}}}{\sqrt{J_{M}}}K(\frac{\sqrt{J_{M}}}{w\sqrt{\mathbf{M}}}h^{\varepsilon })\). Correspondingly, we can also define \({\bar{K}}_i, (i=1,2)\) and kernels \({\bar{k}}_i\) as follows:

$$\begin{aligned} {\bar{K}}_i(f)=\int _{{\mathbb {R}}^3}dq {\bar{k}}_i(p,q)f(q)=\int _{{\mathbb {R}}^3}dq \frac{w(|p|)\sqrt{J_{M}(q)}\sqrt{\mathbf{M}(p)}}{w(|q|)\sqrt{J_{M}(p)}\sqrt{\mathbf{M}(q)}}k_i(p,q)f(q),\qquad i=1,2. \end{aligned}$$

Namely, \({\bar{k}}_i(p,q)= \frac{w(|p|)\sqrt{J_{M}(q)}\sqrt{\mathbf{M}(p)}}{w(|q|)\sqrt{J_{M}(p)}\sqrt{\mathbf{M}(q)}}k_i(p,q), i=1, 2\). Then \({\bar{k}}_i(p,q)\) also satisfy the same estimates in Lemma 8.1.

Corollary 8.1

For \(i=1,2,3\), it holds that

$$\begin{aligned} {\bar{k}}_2(p,q)\lesssim \frac{1}{p^0|p-q|}\exp \Big \{-\frac{c_0\sqrt{s}|p-q|}{T_0g}\Big \}, \end{aligned}$$
(8.6)

for some small constant \(c_0\), and

$$\begin{aligned}&\int _{{\mathbb {R}}^3}dq {\bar{k}}_2(p,q)\lesssim \frac{1}{p^0},\nonumber \\&\quad \int _{{\mathbb {R}}^3}dq \partial _{x_i} {\bar{k}}_2(p,q)\lesssim \frac{1}{p^0},\nonumber \\&\quad \int _{{\mathbb {R}}^3}dq \partial _{p_i} {\bar{k}}_2(p,q)\lesssim \frac{1}{p^0}. \end{aligned}$$
(8.7)

\({\bar{k}}_1(p,q)\) also satisfies the same estimates.

Proof

In fact, we only need to note that

$$\begin{aligned} {\bar{k}}_2(p,q)\le&\frac{(1+|p|)^{\beta }}{(1+|q|)^{\beta }}\frac{s^{\frac{3}{2}}}{gp^0q^0}U_1(p,q)\\&\times \exp \Big \{-[1+O(1)|u|]\frac{\sqrt{s}|p-q|}{2T_0g}+\frac{(T_0-T_M)(p^0-q^0)}{2T_MT_0}\Big \}\\ \lesssim&\frac{s^{\frac{3}{2}}}{gp^0q^0}U_1(p,q) (1+|p-q|)^{\beta }\exp \Big \{[(T_0-2T_M)+O(1)|u|]\frac{\sqrt{s}|p-q|}{2T_0T_Mg}\}\\ \lesssim&\frac{s^{\frac{3}{2}}}{gp^0q^0}U_1(p,q) \exp \Big \{-c_0\frac{\sqrt{s}|p-q|}{T_0T_Mg}\} \end{aligned}$$

by (1.24). Then, similar to the proof of Lemma 8.1, we can obtain (8.6) and (8.7). \(\quad \square \)

1.3 Appendix 3: Construction and estimates of coefficients

We will present the existence of coefficients \(F_n, (1\le n\le 2k-1)\), and their momentum and time regularities estimates. For \(i\in [1, 2k-1]\), we decompose \(\frac{F_n}{\sqrt{{\mathbf {M}}}}\) as the sum of macroscopic and microscopic parts:

$$\begin{aligned} \begin{aligned} \frac{F_n}{\sqrt{{\mathbf {M}}}}&=\mathbf{P}\Big (\frac{F_n}{\sqrt{{\mathbf {M}}}}\Big )+{\{\mathbf{I}-\mathbf{P}\}}\Big (\frac{F_n}{\sqrt{{\mathbf {M}}}}\Big )\\&=[a_n(t,x)+b_n(t,x)\cdot p+c_n(t,x) p^0]\sqrt{{\mathbf {M}}}+{\{\mathbf{I}-\mathbf{P}\}}\Big (\frac{F_n}{\sqrt{{\mathbf {M}}}}\Big ). \end{aligned} \end{aligned}$$

To obtain the linear system satisfied by the abstract functions \(a_{n}(t,x), b_{n}(t,x), c_{n}(t,x)\) and \(E_{n}(t,x)\), \(B_{n}(t,x)\), we first derive the explicit expression of the third momentum \(T^{\alpha \beta \gamma }[{\mathbf {M}}]\), \((\alpha ,\beta ,\gamma \in \{0, 1, 2, 3\})\) :

$$\begin{aligned} T^{\alpha \beta \gamma }[{\mathbf {M}}] = \int _{{\mathbb {R}}^3} \frac{ p^\alpha p^\beta p^{\gamma } }{p^0} {\mathbf {M}}\, d p. \end{aligned}$$

We will first get the expression of \(T^{\alpha \beta \gamma }[\mathbf {M}]\) in the rest frame where \((u^0, u^1, u^2, u^3)=(1, 0, 0, 0)\). For convenience, we denote \(T^{\alpha \beta \gamma }[{\mathbf {M}}]\) as \({\bar{T}}^{\alpha \beta \gamma }\) in the rest frame, p as \({\bar{p}}\) in the rest frame, and \(-v\) as the velocity of general reference frame relative to the rest frame. Then, the corresponding boost matrix \({\bar{\Lambda }}\) is

$$\begin{aligned} {\bar{\Lambda }}=({\bar{\Lambda }}^{\mu }_{\nu })=\left( \begin{array}{cccc} \displaystyle {\tilde{r}} &{} {\tilde{r}}v_1 &{} {\tilde{r}}v_2 &{} {\tilde{r}}v_3\\ \displaystyle {\tilde{r}}v_1 &{} 1+({\tilde{r}}-1)\frac{v_1^2}{|v|^2} &{} ({\tilde{r}}-1)\frac{v_1v_2}{|v|^2} &{} ({\tilde{r}}-1)\frac{v_1v_3}{|v|^2}\\ \displaystyle {\tilde{r}}v_2 &{} ({\tilde{r}}-1)\frac{v_1v_2}{|v|^2} &{} 1+({\tilde{r}}-1)\frac{v_2^2}{|v|^2} &{} ({\tilde{r}}-1)\frac{v_2v_3}{|v|^2}\\ \displaystyle {\tilde{r}}v_3 &{} ({\tilde{r}}-1)\frac{v_1v_3}{|v|^2} &{} ({\tilde{r}}-1)\frac{v_2v_3}{|v|^2} &{}1+({\tilde{r}}-1)\frac{v_3^2}{|v|^2} \end{array} \right) , \end{aligned}$$

where \({\tilde{r}}=u^0, v_i=\frac{u_i}{u^0}\). Noting

$$\begin{aligned} p^{\mu }= {\bar{\Lambda }}^{\mu }_{\nu }{\bar{p}}^{\nu }, \qquad \frac{dp}{p^0}=\frac{d{\bar{p}}}{{\bar{p}}^0}, \end{aligned}$$

we have

$$\begin{aligned} T^{\alpha \beta \gamma }[{\mathbf {M}}]={\bar{\Lambda }}^{\alpha }_{\alpha '}{\bar{\Lambda }}^{\beta }_{\beta '}{\bar{\Lambda }}^{\gamma }_{\gamma '}{\bar{T}}^{\alpha ' \beta '\gamma '}. \end{aligned}$$
(8.8)

Then, we can obtain the expression of \(T^{\alpha \beta \gamma }[{\mathbf {M}}]\) by the expression of \({\bar{T}}^{\alpha \beta \gamma }\) and (8.8). Now we give the expression of \({\bar{T}}^{\alpha \beta \gamma }\) as follows:

Lemma 8.2

Let \(i, j, k\in \{1, 2, 3\}\). For the third momentum \({\bar{T}}^{\alpha \beta \gamma }\) which corresponds to \(T^{\alpha \beta \gamma }[{\mathbf {M}}]\) in the rest frame, we have

$$\begin{aligned}&{\bar{T}}^{000}= \frac{n_0[3K_3(\gamma )+\gamma K_2(\gamma )]}{\gamma K_2(\gamma )}, \end{aligned}$$
(8.9)
$$\begin{aligned}&{\bar{T}}^{0ii}={\bar{T}}^{ii0}={\bar{T}}^{i0i}= \frac{n_0K_3(\gamma )}{\gamma K_2(\gamma )}, \end{aligned}$$
(8.10)
$$\begin{aligned}&{\bar{T}}^{\alpha \beta \gamma }=0, \qquad \text{ if } ~(\alpha ,\beta ,\gamma )\ne (0,0,0), (0,i,i), (i,i,0), (i,0,i). \end{aligned}$$
(8.11)

Proof

It is straightforward to verify (8.11) by the symmetry of the variable of integration. Now we prove (8.9). It holds that

$$\begin{aligned}&{\bar{T}}^{000}= \int _{{\mathbb {R}}^3} (p^0)^2 \frac{n_0 \gamma }{4 \pi K_2(\gamma )} \exp \{- \gamma {\bar{p}}_{0} \} \, d p. \end{aligned}$$

As the proof of Proposition 3.3 in [33], we let \(y=\gamma {\bar{p}}_{0}\) to get \({\bar{p}}^0=\frac{y}{\gamma }\), \(|{\bar{p}}|=\frac{\sqrt{y^2-\gamma ^2}}{\gamma }\), and \(d|{\bar{p}}|=\frac{1 }{\gamma }\frac{y dy}{ \sqrt{y^2-\gamma ^2}}\). Then we have

$$\begin{aligned} {\bar{T}}^{000}&= \int _{\gamma }^{\infty } \frac{1}{\gamma ^2}y^2 \frac{n_0 \gamma }{ K_2(\gamma )} e^{- y} \frac{1}{\gamma ^2}(y^2-\gamma ^2)\frac{1 }{\gamma }\frac{y dy}{ \sqrt{y^2-\gamma ^2}}\\&= \frac{n_0}{\gamma ^4K_2(\gamma )}\int _{\gamma }^{\infty }[(y^2-\gamma ^2)^{\frac{3}{2}}+\gamma ^2\sqrt{y^2-\gamma ^2}]ye^{- y} dy\\&=\frac{n_0[3K_3(\gamma )+\gamma K_2(\gamma )]}{\gamma K_2(\gamma )}. \end{aligned}$$

Similarly, (8.10) can be proved as follows:

$$\begin{aligned} {\bar{T}}^{0ii}&=c \int _{{\mathbb {R}}^3} \frac{|p|^2}{3} \frac{n_0 \gamma }{4 \pi K_2(\gamma )} \exp \{- \gamma {\bar{p}}_{0} \} \, d p\\&= \int _{\gamma }^{\infty } \frac{1}{3\gamma ^2}(y^2-\gamma ^2) \frac{n_0 \gamma }{ K_2(\gamma )} e^{- y} \frac{1}{\gamma ^2}(y^2-\gamma ^2)\frac{1 }{\gamma }\frac{y dy}{ \sqrt{y^2-\gamma ^2}}\\&= \frac{n_0}{3\gamma ^4K_2(\gamma )}\int _{\gamma }^{\infty }(y^2-\gamma ^2)^{\frac{3}{2}}ye^{- y} dy\\&= \frac{n_0K_3(\gamma )}{\gamma K_2(\gamma )}. \end{aligned}$$

\(\square \)

Now we can use Lemma 8.2 and (8.8) to derive the explicit expression of \(T^{\alpha \beta \gamma }[{\mathbf {M}}]\).

Lemma 8.3

For \(i, j, k\in \{1, 2, 3\}\), we have

$$\begin{aligned} \begin{aligned}&T^{000}[{\mathbf {M}}]= \frac{n_0}{\gamma K_2(\gamma )}[(3K_3(\gamma )+\gamma K_2(\gamma ))(u^0)^3+3K_3(\gamma )u^0|u|^2],\\&T^{00i}[{\mathbf {M}}]= \frac{n_0}{\gamma K_2(\gamma )}\left[ (5K_3(\gamma )+\gamma K_2(\gamma ))(u^0)^2u_i+K_3(\gamma )|u|^2u_i\right] ,\\&T^{0ij}[{\mathbf {M}}]=\frac{n_0}{\gamma K_2(\gamma )}\left[ (6K_3(\gamma )+\gamma K_2(\gamma ))u^0u_iu_j+\delta _{ij} K_3(\gamma )u^0\right] ,\\&T^{ijk}[{\mathbf {M}}]=\frac{n_0}{\gamma K_2(\gamma )}(6K_3(\gamma )+\gamma K_2(\gamma ))u_iu_ju_k \\&\qquad +\frac{n_0K_3(\gamma )}{\gamma K_2(\gamma )}(u_i\delta _{jk}+u_j\delta _{ik}+u_k\delta _{ij}). \end{aligned} \end{aligned}$$
(8.12)

Proof

We first prove \(T^{000}[{\mathbf {M}}]\) in (8.12). By Lemma 8.2 and (8.8), we have

$$\begin{aligned} T^{000}[{\mathbf {M}}]&= {\bar{\Lambda }}^0_{\alpha }{\bar{\Lambda }}^0_{\beta }{\bar{\Lambda }}^0_{\gamma }{\bar{T}}^{\alpha \beta \gamma }\\&=\sum _{\alpha =0}{\tilde{r}}\Big ({\tilde{r}}^2\frac{n_0[3K_3(\gamma )+\gamma K_2(\gamma )]}{\gamma K_2(\gamma )}+{\tilde{r}}^2|v|^2\frac{n_0K_3(\gamma )}{\gamma K_2(\gamma )}\Big )\\&\quad +\sum _{\alpha =1}^3{\tilde{r}}v_{\alpha }\times 2{\tilde{r}}^2v_{\alpha }\frac{n_0K_3(\gamma )}{\gamma K_2(\gamma )}\\&= \frac{n_0}{\gamma K_2(\gamma )}\Big ([3K_3(\gamma )+\gamma K_2(\gamma )](u^0)^3+K_3(\gamma )u^0|u|^2\Big )+\frac{2n_0K_3(\gamma )}{\gamma K_2(\gamma )}u^0|u|^2\\&=\frac{n_0}{\gamma K_2(\gamma )}[(3K_3(\gamma )+\gamma K_2(\gamma ))(u^0)^3+3K_3(\gamma )u^0|u|^2]. \end{aligned}$$

For \(T^{00i}[{\mathbf {M}}]\), we have

$$\begin{aligned} T^{00i}[{\mathbf {M}}]&= {\bar{\Lambda }}^0_{\alpha }{\bar{\Lambda }}^0_{\beta }{\bar{\Lambda }}^i_{\gamma }{\bar{T}}^{\alpha \beta \gamma }\\&=\sum _{\alpha =0}{\tilde{r}}\Big ({\tilde{r}}^2v_{i}\frac{n_0[3K_3(\gamma )+\gamma K_2(\gamma )]}{\gamma K_2(\gamma )}\\&\quad +\sum _{j=1}^3{\tilde{r}}v_{j}\Big (\frac{{\tilde{r}}-1}{|v|^2}v_iv_j+\delta _{ij}\Big )\frac{n_0K_3(\gamma )}{\gamma K_2(\gamma )}\Big )\\&\quad +\sum _{\alpha =1}^3{\tilde{r}}v_{\alpha }\Big [{\tilde{r}}^2v_{\alpha }v_i +{\tilde{r}}\Big (\frac{{\tilde{r}}-1}{|v|^2}v_iv_{\alpha }+\delta _{i\alpha }\Big ) \Big ]\frac{n_0K_3(\gamma )}{\gamma K_2(\gamma )}\\&= \frac{n_0}{\gamma K_2(\gamma )}[4K_3(\gamma )+\gamma K_2(\gamma )](u^0)^2u_i+\frac{n_0K_3(\gamma )}{\gamma K_2(\gamma )}u_i[|u|^2+(u^0)^2]\\&=\frac{n_0}{\gamma K_2(\gamma )}\left[ (5K_3(\gamma )+\gamma K_2(\gamma ))(u^0)^2u_i+K_3(\gamma )|u|^2u_i\right] . \end{aligned}$$

Similarly, we obtain

$$\begin{aligned} T^{0ij}[{\mathbf {M}}]&= {\bar{\Lambda }}^0_{\alpha }{\bar{\Lambda }}^i_{\beta }{\bar{\Lambda }}^j_{\gamma }{\bar{T}}^{\alpha \beta \gamma }\\ =&\sum _{\alpha =0}{\tilde{r}}\Big ({\tilde{r}}^2v_{i}v_{j}\frac{n_0[3K_3(\gamma )+\gamma K_2(\gamma )]}{\gamma K_2(\gamma )}\\&+\sum _{k=1}^3\Big (\frac{{\tilde{r}}-1}{|v|^2}v_iv_k+\delta _{ik}\Big )\Big (\frac{{\tilde{r}}-1}{|v|^2}v_jv_k+\delta _{jk}\Big )\frac{n_0K_3(\gamma )}{\gamma K_2(\gamma )}\Big )\\&+\sum _{\alpha =1}^3{\tilde{r}}v_{\alpha }\Big [{\tilde{r}}\frac{v_{i}}{c}\Big (\frac{{\tilde{r}}-1}{|v|^2}v_jv_{\alpha }+\delta _{j\alpha }\Big ) +{\tilde{r}}v_{j}\Big (\frac{{\tilde{r}}-1}{|v|^2}v_iv_{\alpha }+\delta _{i\alpha }\Big )\Big ]\frac{n_0K_3(\gamma )}{\gamma K_2(\gamma )}\\&= \frac{n_0}{\gamma K_2(\gamma )}\{[4K_3(\gamma )+\gamma K_2(\gamma )]u^0u_iu_j+\delta _{ij}K_3(\gamma )u^0\}+2\frac{n_0K_3(\gamma )}{\gamma K_2(\gamma )}u^0u_iu_j\\&=\frac{n_0}{\gamma K_2(\gamma )}\left[ (6K_3(\gamma )+\gamma K_2(\gamma ))u^0u_iu_j+\delta _{ij}K_3(\gamma )u^0\right] , \end{aligned}$$

and

$$\begin{aligned} T^{ijk}[{\mathbf {M}}]&= {\bar{\Lambda }}^i_{\alpha }{\bar{\Lambda }}^j_{\beta }{\bar{\Lambda }}^k_{\gamma }{\bar{T}}^{\alpha \beta \gamma }\\&=\sum _{\alpha =0}{\tilde{r}}v_{i}\Big ({\tilde{r}}^2v_{j}v_{k}\frac{n_0[3K_3(\gamma )+\gamma K_2(\gamma )]}{\gamma K_2(\gamma )}\\&\quad +\sum _{l=1}^3\Big (\frac{{\tilde{r}}-1}{|v|^2}v_jv_l+\delta _{jl}\Big )\Big (\frac{{\tilde{r}}-1}{|v|^2}v_kv_l+\delta _{kl}\Big ) \frac{n_0K_3(\gamma )}{\gamma K_2(\gamma )}\Big )\\&\quad +\sum _{\alpha =1}^3\Big (\frac{{\tilde{r}}-1}{|v|^2}v_iv_{\alpha }+\delta _{i\alpha }\Big ) \Big [{\tilde{r}}v_{j}\Big (\frac{{\tilde{r}}-1}{|v|^2}v_kv_{\alpha }+\delta _{k\alpha }\Big )\\&\quad +{\tilde{r}}v_{k}\Big (\frac{{\tilde{r}}-1}{|v|^2}v_jv_{\alpha }+\delta _{j\alpha }\Big )\Big ]\frac{n_0K_3(\gamma )}{\gamma K_2(\gamma )}\\&= \frac{n_0}{\gamma K_2(\gamma )}\{[3K_3(\gamma )+\gamma K_2(\gamma )]u_iu_ju_k+u_i(u_ju_k+\delta _{jk})K_3(\gamma )\}\\&\quad +\frac{n_0K_3(\gamma )}{\gamma K_2(\gamma )}[u_j(u_iu_k+\delta _{ik})+u_k(u_iu_j+\delta _{ij})]\\&=\frac{n_0}{\gamma K_2(\gamma )}[6K_3(\gamma )+\gamma K_2(\gamma )]u_iu_ju_k \\&\quad +\frac{n_0K_3(\gamma )}{\gamma K_2(\gamma )}(u_i\delta _{jk}+u_j\delta _{ik}+u_k\delta _{ij}). \end{aligned}$$

\(\square \)

With the preparation, now we construct the coefficients \((F_n, E_n, B_n), 1\le n\le 2k-1\) in a conductive way, and estimate their regularities.

Theorem 8.1

For any \(n\in [0,2k-2]\), assume that \((F_i, E_i, B_i)\) have been constructed for all \(0\le i\le n\). Then the microscopic part \({\{\mathbf{I}-\mathbf{P}\}}\Big (\frac{F_{n+1}}{\sqrt{{\mathbf {M}}}}\Big )\) can be written as:

$$\begin{aligned} \begin{aligned} {\{\mathbf{I}-\mathbf{P}\}}\Big (\frac{F_{n+1}}{\sqrt{{\mathbf {M}}}}\Big )&=L^{-1}\Big [-\frac{1}{\sqrt{{\mathbf {M}}}}\Big (\partial _tF_n +{\hat{p}}\cdot \nabla _xF_n-\sum _{\begin{array}{c} i+j=n+1\\ i,j\ge 1 \end{array}}Q(F_i,F_j)\\&-\sum _{\begin{array}{c} i+j=n\\ i,j\ge 0 \end{array}}\Big (E_i+{\hat{p}} \times B_i \Big )\cdot \nabla _pF_j\Big )\Big ]. \end{aligned} \end{aligned}$$

And \(a_{n+1}(t,x), b_{n+1}(t,x), c_{n+1}(t,x)\), \(E_{n+1}(t,x), B_{n+1}(t,x)\) satisfy the following system:

$$\begin{aligned}&\partial _t\Big (n_0u^0a_{n+1}+(e_0+P_0)u^0(u\cdot b_{n+1})+[e_0(u^0)^2+P_0|u|^2]c_{n+1}\Big )\nonumber \\&\qquad +\nabla _x\cdot \Big (n_0u a_{n+1}+(e_0+P_0)u (u\cdot b_{n+1})+P_0 b_{n+1}+(e_0+P_0)u^0 u c_{n+1}\Big )\nonumber \\&\qquad +\nabla _x\cdot \int _{{\mathbb {R}}^3} \frac{p}{p^0}\sqrt{{\mathbf {M}}}{\{\mathbf{I}-\mathbf{P}\}}\Big (\frac{F_{n+1}}{\sqrt{{\mathbf {M}}}}\Big )\,dp=0, \end{aligned}$$
(8.13)
$$\begin{aligned}&\partial _t\Big ((e_0+P_0)u^0u_j a_{n+1}+\frac{n_0}{\gamma K_2(\gamma )}[(6K_3(\gamma )+\gamma K_2(\gamma ))u^0 u_j (u\cdot b_{n+1})\nonumber \\&\qquad +K_3(\gamma )u^0 b_{n+1,j}]+\frac{n_0}{\gamma K_2(\gamma )}[(5K_3(\gamma )+\gamma K_2(\gamma ))(u^0)^2 +K_3(\gamma )|u|^2]u_jc_{n+1}\Big )\nonumber \\&\qquad +\nabla _x\cdot \Big ((e_0+P_0)u_j u a_{n+1}+\frac{n_0}{\gamma K_2(\gamma )}(6K_3(\gamma )+\gamma K_2(\gamma ))u_j u [(u\cdot b_{n+1})+u^0c_{n+1}]\Big )\nonumber \\&\qquad +\partial _{x_j}(P_0a_{n+1})+\nabla _x\cdot \Big [\frac{n_0K_3(\gamma )}{\gamma K_2(\gamma )}(ub_{n+1,j}+u_jb_{n+1})\Big ]\nonumber \\&\qquad + \partial _{x_j}\Big (\frac{n_0K_3(\gamma )}{\gamma K_2(\gamma )}[(u\cdot b_{n+1})+u^0c_{n+1}]\Big )\nonumber \\&\qquad +E_{0,j}\Big (n_0u^0a_{n+1}+(e_0+P_0)u^0(u\cdot b_{n+1})+[e_0(u^0)^2+P_0|u|^2]c_{n+1}\Big )\nonumber \\&\qquad +\Big [\Big (n_0u a_{n+1}+(e_0+P_0)u (u\cdot b_{n+1})+P_0 b_{n+1}+(e_0+P_0)u^0 u c_{n+1}\Big )\times B_0\Big ]_j\nonumber \\&\qquad +n_0u^0E_{n+1,j}+\Big (n_0u\times B_{n+1}\Big )_j\nonumber \\&\qquad +\sum _{\begin{array}{c} k+l=n+1\\ k,l\ge 1 \end{array}}E_{k,j}\Big (n_0u^0a_{l}+(e_0+P_0)u^0(u\cdot b_{l})+P_0b_{l}+[e_0(u^0)^2+P_0|u|^2]c_{l}\Big )\nonumber \\&\qquad +\sum _{\begin{array}{c} k+l=n+1\\ k,l\ge 1 \end{array}}\Big [\Big (n_0u a_{l}+(e_0+P_0)u (u\cdot b_{l})+(e_0+P_0)u^0 u c_{l}\Big )\times B_k\Big ]_j\nonumber \\&\qquad +\nabla _x\cdot \int _{{\mathbb {R}}^3} \frac{p_jp}{p^0}\sqrt{{\mathbf {M}}}{\{\mathbf{I}-\mathbf{P}\}}\Big (\frac{F_{n+1}}{\sqrt{{\mathbf {M}}}}\Big )\,dp+\Big [\int _{{\mathbb {R}}^3} {\hat{p}}\times B_0\sqrt{{\mathbf {M}}}{\{\mathbf{I}-\mathbf{P}\}}\Big (\frac{F_{n+1}}{\sqrt{{\mathbf {M}}}}\Big )\,dp\Big ]_j\nonumber \\&\qquad +\sum _{\begin{array}{c} k+l=n+1\\ k,l\ge 1 \end{array}}\Big [\int _{{\mathbb {R}}^3} {\hat{p}}\times B_k\sqrt{{\mathbf {M}}}{\{\mathbf{I}-\mathbf{P}\}}\Big (\frac{F_{l}}{\sqrt{{\mathbf {M}}}}\Big )\,dp\Big ]_j=0, \end{aligned}$$
(8.14)

for \(j=1, 2, 3\) with \(b_{n+1}=(b_{n+1,1}, b_{n+1,2}, b_{n+1,3})\), \(E_{n+1}=(E_{n+1,1}, E_{n+1,2}, E_{n+1,3})\),

$$\begin{aligned}&\partial _t\Big ([e_0(u^0)^2+P_0|u|^2] a_{n+1}+\frac{n_0}{\gamma K_2(\gamma )}[(5K_3(\gamma )+\gamma K_2(\gamma )) (u^0)^2+K_3(\gamma )|u|^2]\nonumber \\&\qquad \times (u\cdot b_{n+1})+\frac{n_0}{\gamma K_2(\gamma )}[(3K_3(\gamma )+\gamma K_2(\gamma ))(u^0)^2 +3K_3(\gamma )|u|^2]u^0c_{n+1}\Big )\nonumber \\&\qquad +\nabla _x\cdot \Big ((e_0+P_0)u^0 u a_{n+1}+\frac{n_0}{\gamma K_2(\gamma )}(6K_3(\gamma )+\gamma K_2(\gamma ))u^0 u(u\cdot b_{n+1})\nonumber \\&\qquad +\frac{n_0K_3(\gamma )}{\gamma K_2(\gamma )}u^0b_{n+1}+\frac{n_0}{\gamma K_2(\gamma )}[(5K_3(\gamma )+\gamma K_2(\gamma ))(u^0)^2\nonumber \\&\qquad +K_3(\gamma )|u|^2]uc_{n+1} \Big )+n_0u\cdot E_{n+1}+ n_0u\cdot E_0 a_{n+1}\nonumber \\&\qquad +(e_0+P_0)(u\cdot b_{n+1})(u\cdot E_0)+P_0E_0\cdot b_{n+1}\nonumber \\&\qquad +(e_0+P_0)u^0(u\cdot E_0)c_{n+1}+\int _{{\mathbb {R}}^3} {\hat{p}}\sqrt{{\mathbf {M}}}{\{\mathbf{I}-\mathbf{P}\}}\Big (\frac{F_{n+1}}{\sqrt{{\mathbf {M}}}}\Big )\,dp \cdot E_0\nonumber \\&\qquad + \sum _{\begin{array}{c} k+l=n+1\\ k,l\ge 1 \end{array}}\Big [n_0u\cdot E_k a_{l}+(e_0+P_0)(u\cdot b_{l})(u\cdot E_k)+P_0E_k\cdot b_{l}\nonumber \\&\qquad +(e_0+P_0)u^0(u\cdot E_k)c_{l}+\int _{{\mathbb {R}}^3} {\hat{p}}\sqrt{{\mathbf {M}}}{\{\mathbf{I}-\mathbf{P}\}}\Big (\frac{F_{l}}{\sqrt{{\mathbf {M}}}}\Big )\,dp \cdot E_k\Big ]=0, \end{aligned}$$
(8.15)
$$\begin{aligned}&\partial _tE_{n+1}-\nabla _x \times B_{n+1} \nonumber \\&\quad =n_0u a_{n+1}+(e_0+P_0)u (u\cdot b_{n+1})\nonumber \\&\qquad +(e_0+P_0)u^0 u c_{n+1}+\int _{{\mathbb {R}}^3} {\hat{p}}\sqrt{{\mathbf {M}}}{\{\mathbf{I}-\mathbf{P}\}}\Big (\frac{F_{n+1}}{\sqrt{{\mathbf {M}}}}\Big )\,dp, \nonumber \\&\qquad \partial _t B_{n+1}+ \nabla _x \times E_{n+1}=0,\nonumber \\&\qquad \nabla _x\cdot E_{n+1}=-\Big (n_0u^0a_{n+1}+(e_0+P_0)u^0(u\cdot b_{n+1})\nonumber \\&\qquad +[e_0(u^0)^2+P_0|u|^2]c_{n+1}\Big ), \nonumber \\&\nabla _x\cdot B_{n+1}=0. \end{aligned}$$
(8.16)

Furthermore, assume \(a_{n+1}(0,x), b_{n+1}(0,x), c_{n+1}(0,x), E_{n+1}(0,x)\), \(B_{n+1}(0,x)\in H^N, N\ge 0\) be given initial data to the system consisted of equations (8.13), (8.14), (8.15) and (8.16). Then the linear system is well-posed in \(C^0([0,\infty );H^N)\). Moreover, it holds that

$$\begin{aligned}&|F_{n+1}|\lesssim (1+t)^{n}{\mathbf {M}}^{1_-},\qquad |\nabla _pF_{n+1}|\lesssim (1+t)^{n}{\mathbf {M}}^{1_-},\nonumber \\&\quad |\nabla _xF_{n+1}|\lesssim (1+t)^{n}{\mathbf {M}}^{1_-},\qquad |\nabla _p^2F_{n+1}|\lesssim (1+t)^{n}{\mathbf {M}}^{1_-},\nonumber \\&\quad |\nabla _x\nabla _pF_{n+1}|\lesssim (1+t)^{n}{\mathbf {M}}^{1_-},\nonumber \\&\quad |E_{n+1}|+|B_{n+1}|+|\nabla _xE_{n+1}|+|\nabla _xE_{n+1}|\lesssim (1+t)^{n}. \end{aligned}$$
(8.17)

Proof

From the equation of \(F_n\) in (1.10), the microscopic part \({\{\mathbf{I}-\mathbf{P}\}}\Big (\frac{F_{n+1}}{\sqrt{{\mathbf {M}}}}\Big )\) can be written as

$$\begin{aligned} \begin{aligned} {\{\mathbf{I}-\mathbf{P}\}}\Big (\frac{F_{n+1}}{\sqrt{{\mathbf {M}}}}\Big )&=L^{-1}\Big [-\frac{1}{\sqrt{{\mathbf {M}}}}\Big (\partial _tF_n+{\hat{p}}\cdot \nabla _xF_n-\sum _{\begin{array}{c} i+j=n+1\\ i,j\ge 1 \end{array}}Q(F_i,F_j)\\&\qquad -\sum _{\begin{array}{c} i+j=n\\ i,j\ge 0 \end{array}}\Big (E_i+{\hat{p}} \times B_i \Big )\cdot \nabla _pF_j\Big )\Big ]. \end{aligned} \end{aligned}$$

We first prove the equation (8.13). Note that, from (1.14),

$$\begin{aligned} \int _{{\mathbb {R}}^3} F_{n+1} dp&=\int _{{\mathbb {R}}^3} [a_{n+1}+b_{n+1}\cdot p+c_{n+1} p_0]{\mathbf {M}} dp\nonumber \\&=n_0u^0a_{n+1}+(e_0+P_0)u^0(u\cdot b_{n+1})+[e_0(u^0)^2+P_0|u|^2]c_{n+1},\nonumber \\ \int _{{\mathbb {R}}^3} {\hat{p}}_jF_{n+1}\,dp&=n_0u_j a_{n+1}+(e_0+P_0)u_j (u\cdot b_{n+1})+P_0b_{n+1,j}\nonumber \\&+(e_0+P_0)u^0 u_j c_{n+1}+\int _{{\mathbb {R}}^3} {\hat{p}}_j\sqrt{{\mathbf {M}}}{\{\mathbf{I}-\mathbf{P}\}}\Big (\frac{F_{n+1}}{\sqrt{{\mathbf {M}}}}\Big )\,dp, \end{aligned}$$
(8.18)

for \(j=1, 2, 3,\) and

$$\begin{aligned} \int _{{\mathbb {R}}^3} \Big (E_{n+1}+{\hat{p}} \times B_{n+1} \Big )\cdot \nabla _pF_0dp=\int _{{\mathbb {R}}^3}\Big (E_0+{\hat{p}} \times B_0 \Big )\cdot \nabla _pF_{n+1} dp=0. \end{aligned}$$

Then, we integrate the equation of \(F_{n+1}\) in (1.10) w.r.t. p to get (8.13).

Now we prove (8.14). From (1.14) and Lemma 8.3, it holds that

$$\begin{aligned}&\int _{{\mathbb {R}}^3} p_j F_{n+1} dp\\&\quad =\int _{{\mathbb {R}}^3} p_j [a_{n+1}+b_{n+1}\cdot p+c_{n+1} p_0]{\mathbf {M}} dp\\&\quad =(e_0+P_0)u^0u_j a_{n+1}+\frac{n_0}{\gamma K_2(\gamma )}[(6K_3(\gamma )+\gamma K_2(\gamma ))u^0 u_j (u\cdot b_{n+1})\\&\qquad +K_3(\gamma )u^0 b_{n+1,j}]+\frac{n_0}{\gamma K_2(\gamma )}[(5K_3(\gamma )+\gamma K_2(\gamma ))(u^0)^2 +K_3(\gamma )|u|^2]u_jc_{n+1},\\&\int _{{\mathbb {R}}^3} \frac{p_jp}{p^0} F_{n+1} dp\\&\quad =\int _{{\mathbb {R}}^3} \frac{p_jp}{p^0} [a_{n+1}+b_{n+1}\cdot p+c_{n+1} p_0]{\mathbf {M}} dp+\int _{{\mathbb {R}}^3} \frac{p_jp}{p^0}\sqrt{{\mathbf {M}}}{\{\mathbf{I}-\mathbf{P}\}}\Big (\frac{F_{n+1}}{\sqrt{{\mathbf {M}}}}\Big )\,dp\\&\quad =(e_0+P_0)u_j u a_{n+1}+\frac{n_0}{\gamma K_2(\gamma )}(6K_3(\gamma )+\gamma K_2(\gamma ))u_j u [(u\cdot b_{n+1})+u^0c_{n+1}]\nonumber \\&\qquad +e_j P_0a_{n+1}+\frac{n_0K_3(\gamma )}{\gamma K_2(\gamma )}(ub_{n+1,j}+u_jb_{n+1})\nonumber \\&\qquad + e_j\frac{n_0K_3(\gamma )}{\gamma K_2(\gamma )}[(u\cdot b_{n+1})+u^0c_{n+1}]+\int _{{\mathbb {R}}^3} \frac{p_jp}{p^0}\sqrt{{\mathbf {M}}}{\{\mathbf{I}-\mathbf{P}\}}\Big (\frac{F_{n+1}}{\sqrt{{\mathbf {M}}}}\Big )\,dp, \end{aligned}$$

where \(e_j, (j=1,2,3),\) are the unit base vectors in \({\mathbb {R}}^3\), and

$$\begin{aligned}&-\int _{{\mathbb {R}}^3} p_j\Big (E_{n+1}+{\hat{p}} \times B_{n+1} \Big )\cdot \nabla _pF_0dp\\&\quad =\int _{{\mathbb {R}}^3}E_{n+1,j}F_{0}dp+\int _{{\mathbb {R}}^3}\Big ({\hat{p}} \times B_{n+1} \Big )_jF_{0} dp \\&\quad =n_0u^0E_{n+1,j}+\Big (n_0u\times B_{n+1}\Big )_j,\\&-\int _{{\mathbb {R}}^3}p_j\Big (E_0+{\hat{p}} \times B_0 \Big )\cdot \nabla _pF_{n+1} dp\\&\quad =\int _{{\mathbb {R}}^3}E_{0,j}F_{n+1}dp+\int _{{\mathbb {R}}^3}\Big ({\hat{p}} \times B_0 \Big )_jF_{n+1} dp\\&\quad =E_{0,j}\Big (n_0u^0a_{n+1}+(e_0+P_0)u^0(u\cdot b_{n+1})+[e_0(u^0)^2+P_0|u|^2]c_{n+1}\Big )\nonumber \\&\qquad +\Big [\Big (n_0u a_{n+1}+(e_0+P_0)u (u\cdot b_{n+1})+P_0b_{n+1}+(e_0+P_0)u^0 u c_{n+1}\Big )\times B_0\Big ]_j\nonumber \\&\qquad +\int _{{\mathbb {R}}^3}\Big ({\hat{p}} \times B_0 \Big )_j\sqrt{{\mathbf {M}}}{\{\mathbf{I}-\mathbf{P}\}}\Big (\frac{F_{n+1}}{\sqrt{{\mathbf {M}}}}\Big ) dp . \end{aligned}$$

Then, we multiply the equation of \(F_{n+1}\) in (1.10) by \(p_j\) and integrate the resulting equation w.r.t. p to get (8.14).

Next, we show that (8.15) holds. By (1.14) and Lemma 8.3, one has

$$\begin{aligned}&\int _{{\mathbb {R}}^3} p_0 F_{n+1} dp\\&\quad =\int _{{\mathbb {R}}^3} p_0 [a_{n+1}+b_{n+1}\cdot p+c_{n+1} p_0]{\mathbf {M}} dp\\&\quad =[e_0(u^0)^2+P_0|u|^2] a_{n+1}+\frac{n_0}{\gamma K_2(\gamma )}[(5K_3(\gamma )+\gamma K_2(\gamma )) (u^0)^2+K_3(\gamma )|u|^2]\\&\qquad \times (u\cdot b_{n+1})+\frac{n_0}{\gamma K_2(\gamma )}[(3K_3(\gamma )+\gamma K_2(\gamma ))(u^0)^2 +3K_3(\gamma )|u|^2]u^0c_{n+1}, \end{aligned}$$

and

$$\begin{aligned}&-\int _{{\mathbb {R}}^3} p^0\Big (E_{n+1}+{\hat{p}} \times B_{n+1} \Big )\cdot \nabla _pF_0dp\\&\quad =\int _{{\mathbb {R}}^3}E_{n+1}\cdot {\hat{p}}F_{0}dp=n_0u\cdot E_{n+1},\\&-\int _{{\mathbb {R}}^3} p^0\Big (E_{0}+{\hat{p}} \times B_0 \Big )\cdot \nabla _pF_{n+1}dp\\&\quad =\int _{{\mathbb {R}}^3} E_{0}\cdot {\hat{p}}F_{n+1}dp\\&\quad = n_0u\cdot E_0 a_{n+1}+(e_0+P_0)(u\cdot b_{n+1})(u\cdot E_0)+P_0E_0\cdot b_{n+1}\\&\qquad +(e_0+P_0)u^0(u\cdot E_0)c_{n+1}+\int _{{\mathbb {R}}^3} {\hat{p}}\sqrt{{\mathbf {M}}}{\{\mathbf{I}-\mathbf{P}\}}\Big (\frac{F_{n+1}}{\sqrt{{\mathbf {M}}}}\Big )\,dp \cdot E_0. \end{aligned}$$

We integrate the equation of \(F_n\) in (1.10) with \(p^0\) over \({\mathbb {R}}^3_p\) to obtain (8.15). Finally, it is straightforward to obtain the Maxwell system (8.16) of \(E_{n+1}, B_{n+1}\) from (8.18).

Now we prove the well-posedness of the system (8.13), (8.14), (8.15) and (8.16). By conditions (1.16) and the equation \( \partial _t(n_0 u^0) + \nabla _x\cdot (n_0 u) =0\) in (1.17), we simplify equations (8.13), (8.14), (8.15) as follows:

$$\begin{aligned}&n_0u^0\partial _t\Big (a_{n+1}+h(u\cdot b_{n+1})+hu^0c_{n+1}\Big )-\partial _t(P_0c_{n+1})\nonumber \\&\qquad +n_0u\cdot \nabla _x\Big ( a_{n+1}+h (u\cdot b_{n+1})+hu^0 c_{n+1}\Big )+\nabla _x\cdot (P_0b_{n+1})\nonumber \\&\qquad +\nabla _x\cdot \int _{{\mathbb {R}}^3} {\hat{p}}\sqrt{{\mathbf {M}}}{\{\mathbf{I}-\mathbf{P}\}}\Big (\frac{F_{n+1}}{\sqrt{{\mathbf {M}}}}\Big )\,dp=0, \end{aligned}$$
(8.19)
$$\begin{aligned}&n_0u^0\partial _t\Big (hu_j a_{n+1}+\frac{1}{\gamma K_2(\gamma )}[(6K_3(\gamma )+\gamma K_2(\gamma )) u_j (u\cdot b_{n+1}+u^0 c_{n+1})\nonumber \\&\qquad +K_3(\gamma ) b_{n+1,j}]\Big )-\partial _t\Big (\frac{n_0K_3(\gamma )}{\gamma K_2(\gamma )}u_jc_{n+1}\Big )\nonumber \\&\qquad +n_0u\cdot \nabla _x\Big (hu_j a_{n+1}+\frac{1}{\gamma K_2(\gamma )}[(6K_3(\gamma )+\gamma K_2(\gamma ))u_j[(u\cdot b_{n+1})+u^0c_{n+1}]\nonumber \\&\qquad +K_3(\gamma ) b_{n+1,j}]\Big )+\partial _{x_j}(P_0a_{n+1})+\nabla _x\cdot \Big [\frac{n_0K_3(\gamma )}{\gamma K_2(\gamma )}u_jb_{n+1}\Big ]\nonumber \\&\qquad + \partial _{x_j}\Big (\frac{n_0K_3(\gamma )}{\gamma K_2(\gamma )}[(u\cdot b_{n+1})+u^0c_{n+1}]\Big )\nonumber \\&\qquad +E_{0,j}\Big (n_0u^0a_{n+1}+(e_0+P_0)u^0(u\cdot b_{n+1})+[e_0(u^0)^2+P_0|u|^2]c_{n+1}\Big )\nonumber \\&\qquad +\Big [\Big (n_0u a_{n+1}+(e_0+P_0)u (u\cdot b_{n+1})+P_0b_{n+1}+(e_0+P_0)u^0 u c_{n+1}\Big )\times B_0\Big ]_j\nonumber \\&\qquad +n_0u^0E_{n+1,j}+\Big (n_0u^0\times B_{n+1}\Big )_j\nonumber \\&\qquad +\sum _{\begin{array}{c} k+l=n+1\\ k,l\ge 1 \end{array}}E_{k,j}\Big (n_0u^0a_{l}+(e_0+P_0)u^0(u\cdot b_{l})+[e_0(u^0)^2+P_0|u|^2]c_{l}\Big )\nonumber \\&\qquad +\sum _{\begin{array}{c} k+l=n+1\\ k,l\ge 1 \end{array}}\Big [\Big (n_0u a_{l}+(e_0+P_0)u (u\cdot b_{l})+P_0b_l+(e_0+P_0)u^0 u c_{l}\Big )\times B_k\Big ]_j\nonumber \\&\qquad +\nabla _x\cdot \int _{{\mathbb {R}}^3} \frac{p_jp}{p^0}\sqrt{{\mathbf {M}}}{\{\mathbf{I}-\mathbf{P}\}}\Big (\frac{F_{n+1}}{\sqrt{{\mathbf {M}}}}\Big )\,dp+\Big [\int _{{\mathbb {R}}^3} {\hat{p}}\times B_0\sqrt{{\mathbf {M}}}{\{\mathbf{I}-\mathbf{P}\}}\Big (\frac{F_{n+1}}{\sqrt{{\mathbf {M}}}}\Big )\,dp\Big ]_j\nonumber \\&\qquad +\sum _{\begin{array}{c} k+l=n+1\\ k,l\ge 1 \end{array}}\Big [\int _{{\mathbb {R}}^3} {\hat{p}}\times B_k\sqrt{{\mathbf {M}}}{\{\mathbf{I}-\mathbf{P}\}}\Big (\frac{F_{l}}{\sqrt{{\mathbf {M}}}}\Big )\,dp\Big ]_j=0, \end{aligned}$$
(8.20)

and

$$\begin{aligned}&n_0u^0\partial _t\Big (hu^0 a_{n+1}+\frac{1}{\gamma K_2(\gamma )}[(6K_3(\gamma )+\gamma K_2(\gamma )) u^0](u\cdot b_{n+1})\nonumber \\&\qquad +\frac{1}{\gamma K_2(\gamma )}[(3K_3(\gamma )+\gamma K_2(\gamma ))(u^0)^2 +3K_3(\gamma )|u|^2]c_{n+1}\Big )\nonumber \\&\qquad -\partial _t(P_0a_{n+1})-\partial _t\Big (\frac{n_0K_3(\gamma )}{\gamma K_2(\gamma )}(u\cdot b_{n+1})\Big )\nonumber \\&\qquad +n_0u\cdot \nabla _x\Big (hu^0 a_{n+1}+\frac{1}{\gamma K_2(\gamma )}(6K_3(\gamma )+\gamma K_2(\gamma ))u^0(u\cdot b_{n+1})\nonumber \\&\qquad +\frac{1}{\gamma K_2(\gamma )}[(3K_3(\gamma )+\gamma K_2(\gamma ))(u^0)^2+3K_3(\gamma )|u|^2]c_{n+1} \Big )\nonumber \\&\qquad +\nabla _x\cdot \Big (\frac{n_0K_3(\gamma )}{\gamma K_2(\gamma )}u^0b_{n+1}\Big )+\nabla _x\cdot \Big (\frac{2n_0K_3(\gamma )}{\gamma K_2(\gamma )}uc_{n+1}\Big )\nonumber \\&\qquad +n_0u\cdot E_{n+1}+ n_0u\cdot E_0 a_{n+1}+(e_0+P_0)(u\cdot b_{n+1})(u\cdot E_0)\nonumber \\&\qquad +P_0E_0\cdot b_{n+1}+(e_0+P_0)u^0(u\cdot E_0)c_{n+1}\nonumber \\&\qquad +\int _{{\mathbb {R}}^3} {\hat{p}}\sqrt{{\mathbf {M}}}{\{\mathbf{I}-\mathbf{P}\}}\Big (\frac{F_{n+1}}{\sqrt{{\mathbf {M}}}}\Big )\,dp \cdot E_0\nonumber \\&\qquad + \sum _{\begin{array}{c} k+l=n+1\\ k,l\ge 1 \end{array}}\Big [n_0u\cdot E_k a_{l}+(e_0+P_0)(u\cdot b_{l})(u\cdot E_k)+P_0E_k\cdot b_{l}\nonumber \\&\qquad +(e_0+P_0)u^0(u\cdot E_k)c_{l}+\int _{{\mathbb {R}}^3} {\hat{p}}\sqrt{{\mathbf {M}}}{\{\mathbf{I}-\mathbf{P}\}}\Big (\frac{F_{l}}{\sqrt{{\mathbf {M}}}}\Big )\,dp \cdot E_k\Big ]=0. \end{aligned}$$
(8.21)

Equations (8.19), (8.20), (8.21) can be further written as:

$$\begin{aligned}&n_0u^0\Big (\partial _ta_{n+1}+h(u\cdot \partial _t b_{n+1})+hu^0\partial _tc_{n+1}\Big )-P_0\partial _tc_{n+1}\nonumber \\&\qquad +n_0u^0\Big (\partial _t\Big (hu\Big )\cdot b_{n+1})+\partial _t(hu^0)c_{n+1}\Big )-(\partial _tP_0)c_{n+1}\nonumber \\&\qquad +n_0u\cdot \Big ( \nabla _xa_{n+1}+h \nabla _xb_{n+1}\cdot u+hu^0 \nabla _x c_{n+1}\Big )+P_0\nabla _x\cdot b_{n+1}\nonumber \\&\qquad +n_0u\cdot \Big ( \nabla _x\Big (h u\Big )\cdot b_{n+1})+\nabla _x\Big (h u^0\Big ) c_{n+1}\Big ) +b_{n+1}\cdot \nabla _xP_0\nonumber \\&\qquad +\nabla _x\cdot \int _{{\mathbb {R}}^3} {\hat{p}}\sqrt{{\mathbf {M}}}{\{\mathbf{I}-\mathbf{P}\}}\Big (\frac{F_{n+1}}{\sqrt{{\mathbf {M}}}}\Big )\,dp=0, \end{aligned}$$
(8.22)
$$\begin{aligned}&n_0u^0\Big (hu_j \partial _ta_{n+1}+\frac{1}{\gamma K_2(\gamma )}[(6K_3(\gamma )+\gamma K_2(\gamma )) u_j (u\cdot \partial _tb_{n+1}+u^0 \partial _tc_{n+1})\nonumber \\&\qquad +K_3(\gamma ) \partial _tb_{n+1,j}]\Big )-\frac{n_0K_3(\gamma )}{\gamma K_2(\gamma )}u_j\partial _tc_{n+1}-\partial _t\Big (\frac{n_0K_3(\gamma )}{\gamma K_2(\gamma )}u_j\Big )c_{n+1}\nonumber \\&\qquad +n_0u^0\Big [\partial _t\Big (hu_j\Big ) a_{n+1}+\partial _t\Big (\frac{m^2(6K_3(\gamma )+\gamma K_2(\gamma ))}{\gamma K_2(\gamma )} u_j u\Big )\cdot b_{n+1}\nonumber \\&\qquad +\partial _t\Big (\frac{1}{\gamma K_2(\gamma )}(6K_3(\gamma )+\gamma K_2(\gamma )) u_ju^0\Big ) c_{n+1}+\partial _t\Big (\frac{K_3(\gamma )}{\gamma K_2(\gamma )}\Big )b_{n+1,j}\Big ]\nonumber \\&\qquad +n_0u\cdot \Big (hu_j \nabla _xa_{n+1}+\frac{1}{\gamma K_2(\gamma )}(6K_3(\gamma )+\gamma K_2(\gamma ))u_j[( \nabla _xb_{n+1}\cdot u)+u^0\nabla _xc_{n+1}]\nonumber \\&\qquad +K_3(\gamma ) \nabla _xb_{n+1,j}]\Big )+P_0\partial _{x_j}a_{n+1}+\partial _{x_j}P_0a_{n+1}\nonumber \\&\qquad +n_0u\cdot \Big [\nabla _x\Big (hu_j\Big ) a_{n+1}+\nabla _x\Big (\frac{1}{\gamma K_2(\gamma )}(6K_3(\gamma )+\gamma K_2(\gamma ))u_ju\Big )\cdot b_{n+1}\nonumber \\&\qquad +\nabla _x\Big (\frac{1}{\gamma K_2(\gamma )}(6K_3(\gamma )+\gamma K_2(\gamma ))u_ju^0\Big )c_{n+1}\nonumber \\&\qquad +\nabla _x\Big (\frac{K_3(\gamma )}{\gamma K_2(\gamma )}\Big ) b_{n+1,j}\Big ]+\frac{n_0K_3(\gamma )}{\gamma K_2(\gamma )}u_j\nabla _x\cdot b_{n+1}\nonumber \\&\qquad + \frac{n_0K_3(\gamma )}{\gamma K_2(\gamma )}[(u\cdot \partial _{x_j}b_{n+1})+u^0\partial _{x_j}c_{n+1}]\nonumber \\&\qquad +\nabla _x\cdot \Big (\frac{n_0K_3(\gamma )}{\gamma K_2(\gamma )}u\Big )b_{n+1,j}+\nabla _x\Big (\frac{n_0K_3(\gamma )}{\gamma K_2(\gamma )}u_j\Big )\cdot b_{n+1}\nonumber \\&\qquad + \partial _{x_j}\Big (\frac{n_0K_3(\gamma )}{\gamma K_2(\gamma )}u\Big )\cdot b_{n+1}+\partial _{x_j}\Big (\frac{n_0K_3(\gamma )}{\gamma K_2(\gamma )}u^0\Big )c_{n+1}\nonumber \\&\qquad +E_{0,j}\Big (n_0u^0a_{n+1}+(e_0+P_0)u^0(u\cdot b_{n+1})+[e_0(u^0)^2+P_0|u|^2]c_{n+1}\Big )\nonumber \\&\qquad +\Big [\Big (n_0u a_{n+1}+(e_0+P_0)u (u\cdot b_{n+1})+(e_0+P_0)u^0 u c_{n+1}\Big )\times B_0\Big ]_j\nonumber \\&\qquad +n_0u^0E_{n+1,j}+\Big (n_0u^0\times B_{n+1}\Big )_j\nonumber \\&\qquad +\sum _{\begin{array}{c} k+l=n+1\\ k,l\ge 1 \end{array}}E_{k,j}\Big (n_0u^0a_{l}+(e_0+P_0)u^0(u\cdot b_{l})+[e_0(u^0)^2+P_0|u|^2]c_{l}\Big )\nonumber \\&\qquad +\sum _{\begin{array}{c} k+l=n+1\\ k,l\ge 1 \end{array}}\Big [\Big (n_0u a_{l}+(e_0+P_0)u (u\cdot b_{l})+(e_0+P_0)u^0 u c_{l}\Big )\times B_k\Big ]_j\nonumber \\&\qquad +\nabla _x\cdot \int _{{\mathbb {R}}^3} \frac{p_jp}{p^0}\sqrt{{\mathbf {M}}}{\{\mathbf{I}-\mathbf{P}\}}\Big (\frac{F_{n+1}}{\sqrt{{\mathbf {M}}}}\Big )\,dp\nonumber \\&\qquad +\Big [\int _{{\mathbb {R}}^3} {\hat{p}}\times B_0\sqrt{{\mathbf {M}}}{\{\mathbf{I}-\mathbf{P}\}}\Big (\frac{F_{n+1}}{\sqrt{{\mathbf {M}}}}\Big )\,dp\Big ]_j\nonumber \\&\qquad +\sum _{\begin{array}{c} k+l=n+1\\ k,l\ge 1 \end{array}}\Big [\int _{{\mathbb {R}}^3} {\hat{p}}\times B_k\sqrt{{\mathbf {M}}}{\{\mathbf{I}-\mathbf{P}\}}\Big (\frac{F_{l}}{\sqrt{{\mathbf {M}}}}\Big )\,dp\Big ]_j=0, \end{aligned}$$
(8.23)

and

$$\begin{aligned}&n_0u^0\Big (hu^0 \partial _ta_{n+1}+\frac{1}{\gamma K_2(\gamma )}[(6K_3(\gamma )+\gamma K_2(\gamma )) u^0](u\cdot \partial _tb_{n+1})\nonumber \\&\qquad +\frac{1}{\gamma K_2(\gamma )}[(3K_3(\gamma )+\gamma K_2(\gamma ))(u^0)^2 +3K_3(\gamma )|u|^2]\partial _tc_{n+1}\Big )\nonumber \\&\qquad + n_0u^0\Big [\partial _t(hu^0)a_{n+1}+\partial _t\Big (\frac{1}{\gamma K_2(\gamma )}[(6K_3(\gamma )+\gamma K_2(\gamma )) u^0]u\Big )\cdot b_{n+1}\nonumber \\&\qquad +\partial _t\Big (\frac{1}{\gamma K_2(\gamma )}[(3K_3(\gamma )+\gamma K_2(\gamma ))(u^0)^2 +3K_3(\gamma )|u|^2]\Big )c_{n+1}\Big ]\nonumber \\&\qquad -P_0\partial _ta_{n+1}-\frac{n_0K_3(\gamma )}{\gamma K_2(\gamma )}(u\cdot \partial _tb_{n+1})\nonumber \\&\qquad -\partial _tP_0a_{n+1}-\partial _t\Big (\frac{n_0K_3(\gamma )}{\gamma K_2(\gamma )}u\Big )\cdot b_{n+1}\nonumber \\&\qquad +n_0u\cdot \Big (hu^0 \nabla _x a_{n+1}+\frac{1}{\gamma K_2(\gamma )}(6K_3(\gamma )+\gamma K_2(\gamma ))u^0( \nabla _xb_{n+1}\cdot u)\nonumber \\&\qquad +\frac{1}{\gamma K_2(\gamma )}[(3K_3(\gamma )+\gamma K_2(\gamma ))(u^0)^2+3K_3(\gamma )|u|^2]\nabla _xc_{n+1} \Big )\nonumber \\&\qquad +n_0u\cdot \Big [\nabla _x\Big (hu^0\Big ) a_{n+1}+\nabla _x\Big (\frac{1}{\gamma K_2(\gamma )}(6K_3(\gamma )+\gamma K_2(\gamma ))u^0u\Big )\cdot b_{n+1}\nonumber \\&\qquad +\nabla _x\Big (\frac{1}{\gamma K_2(\gamma )}[(3K_3(\gamma )+\gamma K_2(\gamma ))(u^0)^2+3K_3(\gamma )|u|^2]\Big )c_{n+1} \Big ]\nonumber \\&\qquad +\Big (\frac{n_0K_3(\gamma )}{\gamma K_2(\gamma )}u^0\Big )\nabla _x\cdot b_{n+1}+\Big (\frac{2n_0K_3(\gamma )}{\gamma K_2(\gamma )}u\Big )\cdot \nabla _x c_{n+1}\nonumber \\&\qquad +\nabla _x\Big (\frac{n_0K_3(\gamma )}{\gamma K_2(\gamma )}u^0\Big )\cdot b_{n+1}+\nabla _x\cdot \Big (\frac{2n_0K_3(\gamma )}{\gamma K_2(\gamma )}u\Big )c_{n+1}+n_0u\cdot E_{n+1}\nonumber \\&\qquad + n_0u\cdot E_0 a_{n+1}+(e_0+P_0)(u\cdot b_{n+1})(u\cdot E_0)+P_0E_0\cdot b_{n+1}\nonumber \\&\qquad +(e_0+P_0)u^0(u\cdot E_0)c_{n+1}+\int _{{\mathbb {R}}^3} {\hat{p}}\sqrt{{\mathbf {M}}}{\{\mathbf{I}-\mathbf{P}\}}\Big (\frac{F_{n+1}}{\sqrt{{\mathbf {M}}}}\Big )\,dp \cdot E_0\nonumber \\&\qquad +\sum _{\begin{array}{c} k+l=n+1\\ k,l\ge 1 \end{array}}\Big [n_0u\cdot E_k a_{l}+(e_0+P_0)(u\cdot b_{l})(u\cdot E_k)+P_0E_k\cdot b_{l}\nonumber \\&\qquad +(e_0+P_0)u^0(u\cdot E_k)c_{l}+\int _{{\mathbb {R}}^3} {\hat{p}}\sqrt{{\mathbf {M}}}{\{\mathbf{I}-\mathbf{P}\}}\Big (\frac{F_{l}}{\sqrt{{\mathbf {M}}}}\Big )\,dp \cdot E_k\Big ]=0. \end{aligned}$$
(8.24)

Now we can write the equations (8.22), (8.3), (8.24) as a linear symmetric hyperbolic system:

$$\begin{aligned} \mathbf{A}_0\partial _t U+\sum _{i=1}^3\mathbf{A}_i\partial _i U +\mathbf{B}_1 U+\mathbf{B}_2{\bar{U}}=S, \end{aligned}$$
(8.25)

where U and \({\bar{U}}\) are

$$\begin{aligned} U=\left( \begin{array}{c} \displaystyle a_{n+1} \\ \displaystyle b_{n+1}\\ \displaystyle c_{n+1} \end{array} \right) ,\qquad {\bar{U}}=\left( \begin{array}{c} \displaystyle E_{n+1} \\ \displaystyle B_{n+1} \end{array} \right) . \end{aligned}$$

For simplicity, denote

$$\begin{aligned} h_1=h_1(t,x)\equiv \frac{n_0}{\gamma K_2(\gamma )} (6K_3(\gamma )+\gamma K_2(\gamma )), \quad h_2=h_2(t,x)\equiv \frac{n_0K_3(\gamma )}{\gamma K_2(\gamma )} . \end{aligned}$$

\(5\times 5\) Matrixes \(\mathbf{A}_0\), \(\mathbf{A}_i, (i=1,2,3)\) in (8.25) are

$$\begin{aligned} \mathbf{A}_0=\left( \begin{array}{ccc} \displaystyle n_0u^0 &{} n_0u^0hu^t &{} [e_0(u^0)^2+P_0|u|^2]\\ \displaystyle n_0u^0h u &{} (h_1u\otimes u+h_2\mathbf{I})u^0 &{} (h_1(u^0)^2-h_2)u\\ \displaystyle [e_0(u^0)^2+P_0|u|^2] &{} (h_1(u^0)^2-h_2 )u^t &{} (h_1(u^0)^2-3h_2 )u^0 \end{array} \right) , \end{aligned}$$

and

$$\begin{aligned} \mathbf{A}_i=\left( \begin{array}{ccc} \displaystyle n_0u_i &{} n_0hu_iu^t+P_0 e_i^t &{} n_0hu^0u_i\\ \displaystyle n_0hu_iu+P_0 e_i &{} h_1u_iu\otimes u+h_2(u_i \mathbf{I}+\tilde{\mathbf{A}}_i) &{} (h_1u_iu+h_2 e_i)u^0\\ \displaystyle n_0hu^0u_i &{} (h_1u_iu^t+h_2 e_i^t)u^0 &{} (h_1(u^0)^2-h_2 )u_i \end{array} \right) , \end{aligned}$$

where \((\cdot )^t\) denotes the transpose of a vector in \({{\mathbb {R}}^3}\), \(\mathbf{I}\) is the \(3\times 3\) identity matrix and \(\tilde{\mathbf{A}}_i\) is a \(3\times 3\) matrix with its components

$$\begin{aligned} (\tilde{\mathbf{A}}_i)_{jk}=\delta _{ij}u_k+\delta _{ik}u_j,\qquad 1\le j,k\le 3. \end{aligned}$$

The components of matrixes \(\mathbf{B}_1\) and \(\mathbf{B}_2\), and the remainder terms S can be written explicitly as functions of \(F_i, E_i, B_i,~ (0\le i\le n\)) and their first order derivatives. For the matrix \(\mathbf{A}_0\), its determinant is

$$\begin{aligned}&n_0^5\left| \begin{array}{ccc} \displaystyle u_0 &{} u^0u^th &{} h(u^0)^2-\frac{P_0}{n_0}\\ \displaystyle \mathbf{0} &{} u^0[h_3u\otimes u+h_4\mathbf{I}] &{} (u^0)^2h_3u\\ \displaystyle 0 &{} (u^0)^2h_3u^t &{}(u^0)^3h_3-u^0h_4+\frac{1}{\gamma ^2u^0} \end{array} \right| \\&=n_0^5(u^0)^6\left| \begin{array}{cc} \displaystyle h_3u\otimes u+h_4\mathbf{I} &{} h_3u\\ \displaystyle h_3u^0u^t &{} u^0h_3-\frac{1}{u^0}h_4+\frac{1}{\gamma ^2(u^0)^3} \end{array} \right| \\&=n_0^5(u^0)^6\left| \begin{array}{cc} \displaystyle h_4\mathbf{I} &{} h_3u\\ \displaystyle \frac{u^t}{ u^0}\Big [h_4-\frac{1}{\gamma ^2(u^0)^2}\Big ]&{} u^0h_3-\frac{1}{u^0}h_4+\frac{1}{\gamma ^2(u^0)^3} \end{array} \right| \\&=n_0^5(u^0)^6\left| \begin{array}{cc} \displaystyle h_4\mathbf{I} &{} h_3u\\ \displaystyle \mathbf{0}^t&{} \Big [\frac{-|u|^2}{ u^0h_4}\Big (h_4-\frac{1}{\gamma ^2(u^0)^2}\Big )+u^0\Big ]h_3-\frac{1}{u^0}h_4+\frac{1}{\gamma ^2(u^0)^3} \end{array} \right| \\&=n_0^5(u^0)^6h_4^3\Big \{\Big [\frac{-|u|^2}{ u^0h_4}\Big (h_4-\frac{1}{\gamma ^2(u^0)^2}\Big )+u^0\Big ]h_3-\frac{1}{u^0}h_4+\frac{c^4}{\gamma ^2(u^0)^3}\Big \}, \end{aligned}$$

where \(\mathbf{0}\) denotes the column vector \((0,0,0)^t\), \(h_3\) and \(h_4\) are functions with the following forms:

$$\begin{aligned} h_3=-\Big (\frac{K_1(\gamma )}{K_2(\gamma )}\Big )^2-\frac{2}{\gamma }\frac{K_1(\gamma )}{K_2(\gamma )}+1+\frac{8}{\gamma ^2},\quad h_4=\frac{K_1(\gamma )}{\gamma K_2(\gamma )}+\frac{4}{\gamma ^2}. \end{aligned}$$

Noting

$$\begin{aligned} -\Big (\frac{K_1(\gamma )}{K_2(\gamma )}\Big )^2-\frac{3}{\gamma }\frac{K_1(\gamma )}{K_2(\gamma )} +1+\frac{3}{\gamma ^2}>0 \end{aligned}$$

by Proposition 6.3 in Appendix 3 [31], we can further obtain

$$\begin{aligned} |\mathbf{A}_0|&>n_0^5(u^0)^6h_4^3\frac{1}{u^0}(h_3-h_4)\\&=n_0^5(u^0)^5h_4^3\Big [-\Big (\frac{K_1(\gamma )}{K_2(\gamma )}\Big )^2-\frac{3}{\gamma }\frac{K_1(\gamma )}{K_2(\gamma )} +1+\frac{4}{\gamma ^2}\Big ]\\&>\frac{n_0^5(u^0)^5h_4^3}{\gamma ^2}. \end{aligned}$$

On the other hand, the system (8.16) can also be written as a linear symmetric hyperbolic system of \((E_{n+1}, B_{n+1})\):

$$\begin{aligned} \partial _t {\bar{U}}+\sum _{i=1}^3{ {\bar{\mathbf{A}}}}_i\partial _i {\bar{U}} +\mathbf{B}U=0, \end{aligned}$$
(8.26)

where \(\mathbf{B}\) is a \(6\times 5\) matrix whose components are functions of \(n_0, u\). Denote \(\mathbf{O}\) as a \(3\times 3\) matrix with all components be 0, and define matrixes:

$$\begin{aligned} {\bar{\mathbf{A}}}_{11}=\left( \begin{array}{ccc} \displaystyle 0 &{} 0 &{} 0\\ \displaystyle 0 &{} 0 &{} 1\\ \displaystyle 0 &{} -1 &{} 0 \end{array} \right) , \quad {\bar{\mathbf{A}}}_{12}=\left( \begin{array}{ccc} \displaystyle 0 &{} 0 &{} 0\\ \displaystyle 0 &{} 0 &{} -1\\ \displaystyle 0 &{} 1 &{} 0 \end{array} \right) ,\quad {\bar{\mathbf{A}}}_{21}=\left( \begin{array}{ccc} \displaystyle 0 &{} 0 &{} -1\\ \displaystyle 0 &{} 0 &{} 0\\ \displaystyle 1 &{} 0 &{} 0 \end{array} \right) , \\ {\bar{\mathbf{A}}_{22}}=\left( \begin{array}{ccc} \displaystyle 0 &{} 0 &{} 1\\ \displaystyle 0 &{} 0 &{} 0\\ \displaystyle -1 &{} 0 &{} 0 \end{array} \right) ,\quad {\bar{\mathbf{A}}}_{31}=\left( \begin{array}{ccc} \displaystyle 0 &{} 1 &{} 0\\ \displaystyle -1 &{} 0 &{} 0\\ \displaystyle 0 &{} 0 &{} 0 \end{array} \right) ,\quad {\bar{\mathbf{A}}}_{32}=\left( \begin{array}{ccc} \displaystyle 0 &{} -1 &{} 0\\ \displaystyle 1 &{} 0 &{} 0\\ \displaystyle 0 &{} 0 &{} 0 \end{array} \right) . \end{aligned}$$

Then \({ {\bar{\mathbf{A}}}}_i\) in (8.26) can be expressed as

$$\begin{aligned} { {\bar{\mathbf{A}}}}_i=\left( \begin{array}{cc} \displaystyle \mathbf{O} &{} { {\bar{\mathbf{A}}}}_{i1}\\ \displaystyle { {\bar{\mathbf{A}}}}_{i2} &{} \mathbf{O} \end{array}\right) . \end{aligned}$$

Combining (8.25) and (8.26), we can obtain the wellposedness of \((a_{n+1}, b_{n+1}, c_{n+1}, E_{n+1}\), \(B_{n+1}) \in C^0([0,\infty );H^N) \) with initial data \(a_{n+1}(0,x), b_{n+1}(0,x), c_{n+1}(0,x), E_{n+1}(0,x), B_{n+1}(0,x)\) \(\in H^N, N\ge 0\) by the standard theorem of linear symmetric system [3] (Chapter 4.2). Moreover, for \(n=0\), one has

$$\begin{aligned} \Vert \partial _t\mathbf{A}_0\Vert _{\infty }+\sum _{i=1}^{3}\Vert \nabla _x\mathbf{A}_i\Vert _{\infty }+\sum _{i=1}^{2}\Vert \mathbf{B}_i\Vert _{\infty }+\Vert S\Vert _{\infty }\lesssim (1+t)^{-\beta _0}. \end{aligned}$$

Therefore, standard energy estimates on systems (8.25) and (8.26) yield

$$\begin{aligned}&\frac{d}{dt}\left( \Vert (a_1(t),b_1(t),c_1(t))\Vert _{H^N}^2+\Vert [E_1(t),B_1(t)]\Vert _{H^N}^2\right) \\&\quad \lesssim (1+t)^{-\beta _0}[\left( \Vert (a_1(t),b_1(t),c_1(t))\Vert _{H^N}^2+\Vert [E_1(t),B_1(t)]\Vert _{H^N}^2\right) \\&\qquad +\left( \Vert (a_1(t),b_1(t),c_1(t))\Vert _{H^N}+\Vert [E_1(t),B_1(t)]\Vert _{H^N}\right) ]. \end{aligned}$$

Namely,

$$\begin{aligned} \Vert (a_1(t),b_1(t),c_1(t))\Vert _{H^N}+\Vert [E_1(t),B_1(t)]\Vert _{H^N}\lesssim 1 \end{aligned}$$
(8.27)

by Grönwall’s inequality. Let \(Lf_1=\nu f_1+K(f_1)=f_2=w_0(p) \mathbf{M}\), where \(w_0(p)\) is any polynomial of p. Note that, by (1.2), (1.21) and (8.1),

$$\begin{aligned} |K(f_1)|\lesssim \int _{{\mathbb {R}}^3}\frac{s^{\frac{3}{2}}}{gp^0q^0}U_1(p,q)\exp \{-[1+O(1)|u|]\frac{\sqrt{s}|p-q|}{2T_0g} \}|f_1(q)| dq, \end{aligned}$$

and \(f_1=\frac{f_2}{\nu }-\frac{K(f_1)}{\nu }\). Then, we can verify that \(|f_1|\lesssim {\mathbf {M}}^{1_-}\). Therefore, it holds that

$$\begin{aligned} {\{\mathbf{I}-\mathbf{P}\}}\Big (\frac{F_{1}}{\sqrt{{\mathbf {M}}}}\Big )\lesssim (1+|p|){\mathbf {M}}^{1_-}. \end{aligned}$$

It is straightforward to obtain (8.17) by the structure of \(F_1\) and (8.27). Assuming (8.17) holds for \(1\le i\le n\), the case \(i=n+1\) for (8.17) holds again by the structure of the equation for \(F_{n+1}\) in (1.10), the induction assumption and similar analysis as the case \(i=1\) ( \(n=0\)). \(\quad \square \)

Rights and permissions

Reprints and permissions

About this article

Check for updates. Verify currency and authenticity via CrossMark

Cite this article

Guo, Y., Xiao, Q. Global Hilbert Expansion for the Relativistic Vlasov–Maxwell–Boltzmann System. Commun. Math. Phys. 384, 341–401 (2021). https://doi.org/10.1007/s00220-021-04079-x

Download citation

  • Received:

  • Accepted:

  • Published:

  • Issue Date:

  • DOI: https://doi.org/10.1007/s00220-021-04079-x

Navigation